SlideShare une entreprise Scribd logo
1  sur  31
CHAPTER 4
                              FOURIER TRANSFORMS
INTEGRAL TRANSFORM
                                                                              b

       The integral transform of a function f (x ) is defined by              ∫ f ( x).k (s , x)dx where
                                                                              a

       k(s , x) is a known function of s and x and it is called the kernel of the transform.
       When k(s , x) is a sine or cosine function, we get transforms called Fourier sine or
cosine transforms.

FOURIER INTEGRAL THEOREM

       If f (x) is a given function defined in (-l , l) and satisfies Dirichlet’s conditions, then
                           ∞ ∞
                             1
                            π ∫ −∫
                f ( x) =           f (t ) cos λ (t − x) dt dλ
                               0 ∞

At a point of discontinuity the value of the integral on the left of above equation is
1
  { f ( x + 0) − f ( x − 0)}.
2

EXAMPLES
                                 1 for x ≤ 1
                                 
1. Express the function f ( x) =                as a Fourier Integral. Hence evaluate
                                 0 for x > 1
                                 
∞                                       ∞
  sin λ cos λx                            sin λ
∫ λ
0
               dλ and find the value of ∫
                                        0
                                            λ
                                                dλ .


Solution:
       We know that the Fourier Integral formula for f (x) is
                              ∞ ∞
                          1
                 f ( x) =
                          π   ∫ ∫ f (t ) cos λ (t − x) dt dλ
                              0 −∞
                                                                                         ……………….(1)

       Here      f (t ) = 1 for      t ≤ 1          i.e.,   f(t) = 1 in -1 < t < 1
                 f (t ) = 0 for t > 1
                 f (t ) = 0 in − ∞ < t < −1 and 1 < t < ∞
                                             ∞ 1
                                         1
∴ Equation (1) ⇒              f ( x) =
                                         π   ∫ ∫ cos λ (t − x) dt dλ
                                             0 −1


                                             ∞                   1
                                         1  sin λ (t − x) 
                                         π ∫
                                     =                     dλ
                                           0
                                                  λ         −1
                                             ∞
                                         1 sin λ (1 − x) − sin λ (−1 − x)
                                         π∫
                                     =                                    dλ
                                          0
                                                          λ
                                             ∞
                                      1 sin λ (1 − x) + sin λ (1 + x )
                                     = ∫                               dλ
                                      π 0             λ


                                                                                                           1
∞
                                             2 sin λ cos λx
                                             π∫
                                ∴ f ( x) =                  dλ                        .………………(2)
                                              0
                                                     λ

                                                      [Using sin (A+B) + sin (A-B) = 2 sin A cos B]

This is Fourier Integral of the given function. From (2) we get

                        ∞
                          sin λ cos λx                 π
                        ∫
                        0
                                λ
                                       dλ =
                                                       2
                                                         f ( x)                      ……………….(3

                                                      
                                                      1 for x ≤ 1
                                  But        f ( x) =                               ………………..(4)
                                                      0 for x > 1
                                                      
Substituting (4) in (3) we get

                                                π
                            ∞
                              sin λ cos λx         for x ≤ 1
                            ∫ λ            dλ =  2
                            0                   0 for x > 1
                                                
                       ∞
                          sin λ      π
Putting x = 0 we get   ∫0
                            λ
                                dλ =
                                     2

2. Find the Fourier Integral of the function
                                  0     x<0
                                  1
                                  
                         f ( x) =        x=0
                                  2
                                  e − x x > 0
                                  
Verify the representation directly at the point x = 0.
Solution:
       The Fourier integral of f (x) is
                                ∞ ∞
                            1
                 f ( x) =
                            π   ∫ ∫ f (t ) cos λ (t − x) dt dλ
                                0 −∞
                                                                                       ……………….(1)

                        1                                                          
                          ∞   0                            ∞
                       =  ∫ −∫∞
                        π 0
                                 f (t ) cos λ (t − x )dt + ∫ f (t ) cos λ (t − x)dt d λ
                                                           0                        
                        1                                                     
                          ∞   0                         ∞
                       = ∫  ∫ 0. cos λ (t − x)dt + ∫ e −t cos λ (t − x)dt dλ
                        π 0 − ∞                        0                      
                          ∞                                                    ∞
                        1  e −t
                       = ∫ 2     [ − cos( λt − λx ) + λ sin(λt − λx)]  dλ
                                                                       
                        π 0 λ +1                                      0

                                  ∞
                                1 cos λx + λ sin λx
                                π∫
                   f (x) =                          dλ                                     ……….………(2)
                                 0     λ2 + 1




                                                                                                      2
Putting x = 0 in (2), we get
                             ∞
                           1
                  f (0) = ∫ 2
                                 1
                          π 0 λ +1
                                           1
                                     d λ = tan −1 ( λ ) 0
                                           π
                                                        ∞
                                                            [           ]
                           1
                                     [
                        = tan −1 ( ∞ ) − tan −1 (0)
                          π
                                                                  ]
                           1 π  1
                        =  =
                          π 2 2
                                             1
The value of the given function at x = 0 is . Hence verified.
                                             2

FOURIER SINE AND COSINE INTEGRALS

       The integral of the form
                             2∞       ∞
                    f ( x) = ∫ sin λx ∫ f (t ) sin λt dt dλ
                            π0        0

is known as Fourier sine integral.

        The integral of the form
                                ∞      ∞
                              2
                     f ( x) = ∫ cos λx ∫ f (t ) cos λt dt dλ
                             π 0       0



is known as Fourier cosine integral.

PROBLEMS

1. Using Fourier integral formula, prove that

                                               2(b 2 − a 2 ) ∞       u sin xu
                                                             ∫ (u 2 + a 2 )(u 2 + b 2 ) du (a, b > 0)
                         − ax        − bx
                     e          −e           =
                                                    π        0



Solution:
       The presence of sin xu in the integral suggests that the Fourier sine integral formula
has been used.
       Fourier sine integral representation is given by

                                         ∞          ∞
                              2
                      f ( x) = ∫ sin ux ∫ f (t ) sin ut dt du
                              π 0       0


                                               ∞
                                                           ∞                               
                                                 sin ux du  ∫ ( e − at − e −bt ) sin ut dt 
                                             2
                    e − ax − e −bx =           ∫
                                             π 0           0                               




                                                                                                        3
∞                                                                                        ∞
                                       2                           e − at                                − bt
                                                                                                                                      
                                      = ∫ sin ux du                2        { − a sin ut − u cos ut} − 2e 2 { − b sin ut − u cos ut} 
                                       π 0                        a + u
                                                                           2
                                                                                                       b +u                           0
                                              ∞
                                           2              u               u 
                                      =      ∫ sin ux du  a 2 + u 2 − b 2 + u 2 
                                           π 0                                  
                                                              ∞
                                        2(b 2 − a 2 )         u sin ux
                                      =
                                             π        ∫ (u 2 + a 2 )(u 2 + b 2 ) du
                                                      0


2. Using Fourier integral formula, prove that

                                        2
                                              ∞
                                                  (λ   2
                                                           + 2 ) cos xλ
                                              ∫                         dλ
                           −x
                       e        cos x =
                                        π     0            λ2 + 4

Solution:
       The presence of cos xλ in the integral suggests that the Fourier cosine integral
formula for e − x cos x has been used.
       Fourier cosine integral representation is given by

                                       ∞               ∞
                                     2
                                     π∫
                       f ( x) =          cos λx ∫ f (t ) cos λt dt dλ
                                       0        0



                              2∞           ∞ − t                
             ∴e   −x
                       cos x = ∫ cos xλ dλ  ∫ e cos t cos λt dt 
                              π 0          0                    

                                     2
                                       ∞
                                                   1 ∞                                      
                                 =     ∫ cos xλ dλ  ∫ e −t { cos(λ + 1)t + cos(λ − 1)t } dt 
                                     π 0           2 0                                      

                                       ∞
                                                    
                                 =
                                     2
                                     π 0            
                                                          1          −t
                                                                             [
                                       ∫ cos xλ dλ  (λ + 1) 2 + 1 e { − cos(λ + 1)t + (λ + 1) sin(λ + 1)t}            ]   ∞
                                                                                                                           0
                                                    
                                          +
                                                  1
                                             (λ − 1) + 1
                                                     2
                                                                    [
                                                         e −t { − cos(λ − 1)t + (λ − 1) sin(λ − 1)t } 0
                                                                                                      ∞
                                                                                                              ]   )
                                     ∞
                                   1         1           1     
                                  = ∫              +            cos xλ dλ
                                   π 0  (λ + 1) + 1 (λ − 1) + 1
                                                2           2




                                          ∞
                                   2 (λ2 + 2) cos xλ
                                  = ∫                dλ.
                                   π 0   λ2 + 4




COMPLEX FORM OF FOURIER INTEGRALS


                                                                                                                           4
The integral of the form

                                ∞              ∞
                            1
                                ∫    e − iλx   ∫ f (t ) e
                                                             iλt
                f ( x) =                                           dt d λ
                           2π   −∞             −∞


is known as Complex form of Fourier Integral.

FOURIER TRANSFORMS
COMPLEX FOURIER TRANSFORMS
                                        ∞
                                    1
         The function F [ f ( x)] =     ∫∞ f (t ).e dt is called the Complex Fourier transform
                                                   ist

                                    2π −
of f (x ) .

INVERSION FORMULA FOR THE COMPLEX FOURIER TRANSFORM
                               ∞
                           1
                               ∫∞F [ f ( x)].e ds is called the inversion formula for the
                                              −isx
     The function f ( x) =
                           2π −
Complex Fourier transform of F [ f ( x)] and it is denoted by F −1 [ F ( f ( x))].

FOURIER SINE TRANSFORMS
                                                        ∞
                                    2
       The function FS [ f ( x )] =                     ∫ f (t ).sin st dt      is called the Fourier Sine Transform of
                                    π                   0

the function f (x ) .

                                               ∞
                             2
       The function f ( x) =
                             π                 ∫ F [ f ( x)]. sin sx ds is called the inversion formula for the
                                               0
                                                    S


Fourier sine transform and it is denoted by FS
                                                                       −1
                                                                            [ FS ( f ( x))].
FOURIER COSINE TRANSFORMS
                                    ∞
                                  2
      The function FC [ f ( x)] =
                                  π ∫
                                      f (t ). cos st dt is called the Fourier Cosine
                                    0

Transform of f (x) .
                                               ∞
                             2
       The function f ( x) =
                             π                 ∫ F [ f ( x)]. cos sx ds
                                               0
                                                    C                               is called the inversion formula for the

Fourier Cosine Transform and it is denoted by FC
                                                                               −1
                                                                                    [ FC ( f ( x))].
PROBLEMS

1. Find the Fourier Transform of
                  1 − x 2 in x ≤ 1
                  
         f ( x) = 
                  0
                          in x > 1




                                                                                                                          5
∞
                                 sin s − s cos s    s     3π
   Hence prove that         ∫
                            0          s 3
                                                 cos ds =
                                                    2     16
                                                             .


Solution:
       We know that the Fourier transform of f (x) is given by

                                             ∞
                                 1
           F [ f ( x )] =                    ∫ f ( x).e
                                                            isx
                                                                  dx
                                 2π      −∞



                                             −1                                   1                                      ∞
                                1                                            1                                  1
                      =                      ∫ f ( x).e           dx +            ∫ f ( x).e         dx +                ∫ f ( x).e
                                                            isx                                isx                                      isx
                                                                                                                                              dx
                                2π       −∞                                  2π   −1                            2π       1
                                         −1                                  1                                      ∞
                                1                                      1                                    1
                     =                   ∫ 0.e dx +                          ∫ (1 − x ).e dx +                      ∫ 0.e
                                              isx                                    2   isx                                 isx
                                                                                                                                   dx
                                2π       −∞                            2π    −1                             2π       1


                                         1
                                1
                     =                   ∫ (1 − x
                                                       2
                                                           ).e isx dx
                                2π       −1



                                                                                                       1
                                1              e isx            e isx   e isx 
                     =              (1 − x 2 )       − ( −2 x ) 2 2 − 2 3 3 
                                2π              is             i s     i s  −1


                                1  − 2 is  2 is − 2 −is 2 e −is 
                     =              2 e + 3e + 2 e −            
                                2π  s     is    s       i s3 


                                1  − 2 is             2                
                     =             s   (e + e −is ) + 3 (e is − e −is )
                                2π 
                                      2
                                                      is                

                                1 − 4           4                                    1 4                       
                     =              s 2 cos s + s 3 sin s  =                             s 3 (sin s − s cos s )
                                2π                                                   2π                        

By using inverse Fourier Transform we get

                                                       ∞
                                1                 1         4
               f ( x) =
                                2π
                                         .
                                                  2π
                                                       ∫
                                                       −∞   s 3
                                                                (sin s − s cos s ).e −isx ds

                                    ∞
                           1             4
                     =
                          2π        ∫s
                                    −∞
                                             3
                                                  (sin s − s cos s ).(cos sx − i sin sx ) ds

                                    ∞
                           1             4
                     =
                          2π        ∫
                                    −∞   s3
                                            (sin s − s cos s ) cos sx ds

                                                                   ∞
                                                          1                4
                                                       −
                                                         2π        ∫
                                                                   −∞      s3
                                                                              (sin s − s cos s ) i sin sx ds



                                                                                                                                                   6
The second integral is odd and hence its values is zero.

                                  ∞
                              2 sin s − s cos s
                              π −∫
          ∴        f ( x) =                     cos sx ds
                                 ∞    s3
                                  ∞
                             4 sin s − s cos s
                            = ∫                cos sx ds
                             π 0     s3

          ∞
            sin s − s cos s            π
 i.e.,    ∫
          0        s 3
                            cos sx ds = f ( x)
                                       4
             1
Putting x = , we get
             2
         ∞
             sin s − s cos s     s     π  1  π  1  3π
         ∫0         s  3
                             cos ds =
                                 2
                                         f   = 1 −  =
                                        4  2  4  4  16
                                                           .

          ∞
              sin s − s cos s    s     3π
          ∫
          0         s 3
                              cos ds =
                                 2     16
                                          .


2. Find the Fourier sine transform of e − x , x ≥ 0 (or) e − x , x > 0. Hence evaluate
    ∞
      x sin mx
    ∫ 1 + x 2 dx.
    0

Solution:
       The Fourier sine transform of f(x) is given by

                                          ∞
                              2
              FS [ f ( x )] =             ∫ f ( x).sin sx dx
                              π           0
         −x        −x
Here e        =e        for x > 0

                                      ∞
                   [ ]
              FS e − x =
                                  2
                                  π   ∫e
                                              −x
                                                   . sin sx dx
                                      0

                              2 s                                ∞ − ax             b 
                            =                                     ∫ e sin bx dx = 2     
                              π s2 +1                            0               a + b2 
Using inverse Fourier sine transform we get

                                  ∞

                                  ∫ F [e ]. sin sx ds
                              2               −x
              f ( x) =                s
                              π   0


                                  ∞
                              2           2     s
                        =
                              π   ∫
                                  0
                                            . 2
                                          π s +1
                                                  . sin sx ds

                              ∞
                            2      s
                        =     ∫ s 2 + 1 sin sx ds
                            π 0


                                                                                             7
∞
                 π              s
      i.e.,        f ( x) = ∫ 2    . sin sx ds
                 2          0 s +1
                ∞
                      s. sin sx     π
      i.e.,     ∫0     s +1
                         2
                                ds = e − x
                                    2

Replacing x by m we get

                         ∞
                             s. sin ms     π
              i.e.,      ∫
                         0    s +1
                                 2
                                       ds = e − m
                                           2

                         ∞
                             x. sin mx     π
              i.e.,      ∫
                         0    x +1
                                 2
                                       dx = e −m
                                           2
                                                                      [since s is dummy variable, we can replace it by x]


                                                                      e − ax
3. Find the Fourier cosine transform of                                      .
                                                                        x
Solution:
                                                                 ∞
                                                             2
              We know that FC [ f ( x )] =                       ∫ f ( x). cos sx dx
                                                             π   0
                                            − ax
                                        e
Here                         f ( x) =              .
                            x
                                                   ∞
                            2                        e − ax
          ∴ FC [ f ( x )] =                        ∫ x . cos sx dx
                            π                      0



Let                   FC [ f ( x)] = F ( s )

                                                   ∞
                                            2        e − ax
Then                         F (s) =
                                            π      ∫ x . cos sx dx
                                                   0
                                                                                                                   ………………(1)


Differentiating on both sides w.r.t. ‘s’ we get,

                                                       ∞
                       dF ( s ) d 2                      e − ax
                        ds
                               =
                                 ds π                  ∫ x . cos sx dx
                                                       0
                                         ∞
                                        2 ∂  e −ax       
                                =        ∫ ∂s  x . cos sxdx
                                        π0               
                                             ∞                                       ∞
                                        2 e −ax                  2 − ax
                                =        ∫ x (− sin sx).x dx = − π ∫ e sin sx dx
                                        π0                         0

                       dF ( s )    2     s                                      ∞
                                                                                                                b 
                                                                                 ∫e
                                                                                         − ax
                                =−   . 2                                                      sin bx dx =          
                        ds         π a + s2                                     0                            a + b2 
                                                                                                               2




Integrating w.r.t. ‘s’ we get


                                                                                                                               8
2      s
            F (s) = −      .∫ 2   ds
                         π s + a2

                        2 1                        1
                 =−      . . log ( s 2 + a 2 ) = −    . log ( s 2 + a 2 )
                        π 2                        2π

                                                                 e − ax − e −bx
4. Find the Fourier cosine transform of                                         .
                                                                        x
Solution:
       We know that the Fourier cosine transform of f(x) is

                                            ∞
                                      2
               FC [ f ( x )] =              ∫ f ( x). cos sx dx
                                      π         0
                                     − ax
                                 e          − e −bx
Here                 f ( x) =
                                            x

                                            ∞
         e − ax − e −bx   2                 e − ax − e − bx
   ∴ FC 
                x
                         =
                           π               ∫ x . cos sx dx
                                            0


                                            ∞                                  ∞
                             2                e − ax          2 e −bx
                           =
                             π              ∫ x . cos sx dx − π ∫ x cos sx dx
                                            0                   0




                                 e −ax        e −bx 
                           = Fc         − Fc        
                                 x            x 

                                            1                             1
                           =−                       log ( s 2 + a 2 ) +            log ( s 2 + b 2 )
                                            2π                            2π


                                      1       s2 + b2 
                           =             log  2
                                              s + a2 
                                      2π              

                                                            e − as
5. Find f (x) , if its sine transform is                           . Hence deduce that the inverse sine
                                                              s
               1
transform of     .
               s
Solution:
       We know that the inverse Fourier sine transform of FS [ f (x )] is given by
                                                ∞
                              2
                     f ( x) =
                              π                 ∫ F [ f ( x)]. sin sx ds
                                                0
                                                    S


                                 e − as
Here           FS [ f ( x )] =
                                   s


                                                                                                          9
∞
                                      2     e − as
            ∴          f ( x) =
                                      π   ∫ s . sin sx ds
                                          0



Differentiating w.r.t. ‘x’ on both sides, we get,

                 d [ f ( x )]
                                              ∞
                                2               e − as ∂
                     dx
                              =
                                π             ∫ s . ∂x (sin sx) ds
                                              0
                                               ∞                                    ∞
                                          2      e − as            2 − as
                                  =
                                          π    ∫ s . cos sx s ds = π ∫ e . cos sx ds
                                               0                     0

                                          2    a                   ∞ −ax              a 
                                  =                                ∫ e cos bx dx = 2     
                                          π a + x2
                                             2
                                                                   0               a + b2 
                  d [ f ( x )]   2    a
                               =
                      dx         π x + a2
                                    2


                                              2       1            2 1    −1  x 
                  ∴ f ( x) = a
                                              π ∫ x 2 + a 2 dx = a π a tan  a 
                                                                              
                                          2         x
                  ∴ f ( x) =                tan −1  
                                          π        a
                                                                   1
To find the inverse Fourier sine transform of                        :
                                                                   s
           Put a = 0, in (1), we get

                         2              2 π  π
            f ( x) =       tan −1 (∞) =  . =
                         π              π 2  2

PROPERTIES

 1. Linearity Property
       If F(s) and G(s) are the Fourier transform of f (x ) and g (x) respectively then
                 F [ a f ( x ) + b g ( x)] = a F ( s) + b G ( s )
Proof:
                                            ∞
                                       1
        F [ a f ( x) + b g ( x)] =          ∫∞[ a f ( x) + b g ( x)] e dx
                                                                      isx

                                       2π −

                                                   ∞                                     ∞
                                              1                                 1
                                      =            ∫ a f ( x).e dx +                     ∫ b g ( x).e
                                                               isx                                      isx
                                                                                                              dx
                                              2π   −∞                           2π      −∞



                                                   ∞                                ∞
                                              a                             b
                                      =            ∫    f ( x).e isx dx +           ∫ g ( x).e
                                                                                                 isx
                                                                                                       dx
                                              2π   −∞                       2π      −∞



                                      = a F ( s ) + b G ( s)



                                                                                                                   10
2. Change of Scale Property

                                                                                                                         1 s
         If F(s) is the Fourier transform of f (x ) then F [ f (ax)] =                                                    F  ,a>0
                                                                                                                         a a
Proof:
                                           ∞
                                  1
         F [ f (ax)] =                     ∫ f (ax).e
                                                              isx
                                                                     dx
                            2π             −∞
Put                   ax = y
                                                                       dy
                     a dx = dy                 i.e., dx =
                                                                       a
When x = −∞,            y = −∞ and x = ∞,                                   y=∞

                                           ∞                     y                             ∞             s
                                  1                                    dy 1            1                    i  y
         F [ f (ax)] =
                                                            is

                                  2π
                                           ∫ f ( y).e
                                           −∞
                                                                 a
                                                                     .   =
                                                                       a a             2π
                                                                                               ∫ f ( y).e
                                                                                              −∞
                                                                                                             a
                                                                                                                     .dy


                              1
                         =      F ( s a)
                              a

3. Shifting Property ( Shifting in x )

         If F(s) is the Fourier transform of f (x ) then F [ f ( x − a )] = e ias F ( s )
Proof:
                                                 ∞
                                       1
         F [ f ( x − a)] =                       ∫ f ( x − a).e
                                                                            isx
                                                                                  dx
                                       2π       −∞


Put     x-a = y
         dx = dy
When x = −∞, y = −∞ and x = ∞,                                              y=∞

                                                 ∞                                                 ∞
                                       1                                                   e ias
         F [ f ( x − a)] =                       ∫   f ( y ).e is ( y + a ) . dy =                 ∫ f ( y).e
                                                                                                                isy
                                                                                                                         .dy
                                       2π       −∞                                          2π     −∞



                                                ∞
                                  e ias
                              =                 ∫ f ( x).e             .dx = e isa F ( s )
                                                                 isx

                                      2π        −∞


4. Shifting in respect of s

         If F(s) is the Fourier transform of f (x ) then F e iax f ( x) = F ( s + a )               [                ]
Proof:
                                                ∞
         Fe[   iax
                          ]
                     f ( x) =
                                      1
                                                ∫e
                                                     iax
                                                           f ( x) e isx dx
                                      2π       −∞




                                                                                                                                      11
∞
                              1
                                        ∫ f ( x).e
                                                         i( s+a) x
                        =                                            dx = F ( s + a)
                              2π     −∞


5. Modulation Theorem
                                                                                                             1
         If F(s) is the Fourier transform of f (x ) then F [ f ( x) cos ax ] =                                 [ F ( s + a ) + F ( s − a )]
                                                                                                             2
Proof:
                                            ∞
                                    1
         F [ f ( x) cos ax ] =              ∫ f ( x). cos ax.e
                                                                         isx
                                                                               dx
                                    2π      −∞



                                            ∞
                                    1                   e iax + e −iax             
                            =           ∫
                                    2π −∞
                                          f ( x).e isx 
                                                       
                                                              2
                                                                                    dx
                                                                                    
                                                                                    

                                                     ∞                                     ∞
                               1 1                                      1 1
                                                  ∫∞ f ( x).e dx + 2 . 2π                  ∫ f ( x).e
                                                             i( s+a ) x                                 i ( s −a ) x
                              = .                                                                                      dx
                               2 2π              −                                         −∞



                                  1              1             1
                              =     f ( s + a ) + f ( s − a ) = [ f ( s + a ) + f ( s − a )]
                                  2              2             2


                                  1
         F [ f ( x) cos ax ] =      [ F ( s + a ) + F ( s − a)]
                                  2

COROLLARIES
                             1
(i ) FC [ f ( x) cos ax ] =    [ FC ( s + a) + FC ( s − a)]
                             2
                             1
(ii ) FC [ f ( x) sin ax ] = [ FS (a + s) + FS (a − s )]
                             2
                               1
(iii ) FS [ f ( x) cos ax ] = [ FS ( s + a ) + FS ( s − a )]
                               2
                             1
(iv ) FS [ f ( x) sin ax ] = [ FC ( s − a ) − FC ( s + a)]
                              2

6. Conjugate Symmetry Property

         If F(s) is the Fourier transform of f (x ) then F f ( − x) = F ( s)           [        ]
Proof:
                                                         ∞
                                                 1
         We know that F ( s ) =                          ∫ f ( x). e
                                                                         isx
                                           dx
                                 2π −∞
Taking complex conjugate on both sides we get
                     ∞
                 1
                     ∫∞ f ( x). e dx
                                 −isx
         F (s) =
                 2π −


                                                                                                                                         12
Put     x = -y
      dx = -dy
When x = −∞, y = ∞ and x = ∞,                                  y = −∞

                                                −∞
                                       1
                ∴ F (s) =                        ∫ f (− y) .e              (− dy )
                                                                     isy

                                       2π        ∞



                                                      −∞
                                           1
                               =−                     ∫ f (− y). e
                                                                           isy
                                                                                 dy
                                           2π         ∞




                                                                                          [            ]
                                                 ∞
                                       1
                               =                 ∫ f (− x). e              dx = F f (− x)
                                                                     isx

                                       2π       −∞


7. Transform of Derivatives

         If F(s) is the Fourier transform of f (x ) and if f (x) is continuous, f ′(x) is piecewise
continuously differentiable, f (x ) and f ′(x) are absolutely integrable in (−∞ , ∞) and
 lim [ f ( x)] = 0 , then
x → ±∞

                F ( f ′( x ) ) = −is F ( s )
Proof:
         By the first three conditions given, F { f (x)} and F { f ′(x)} exist.
                                            ∞
                                   1
              F { f ′( x)} =                ∫ f ′( x) e
                                                               isx
                                                                     dx
                                   2π      −∞



                                                                                      ∞
                           =
                                   1
                                           [e   isx
                                                           ]   ∞
                                                      f ( x) −∞ −
                                                                             is
                                                                                      ∫e
                                                                                              isx
                                                                                                    f ( x) dx, on int egrating by parts.
                                   2π                                        2π       −∞



                           = 0 − isF { f ( x)} , by the given condition.


                           = −is F ( s).

The theorem can be extended as follows.

       If f , f ′, f ′′, , f ( n −1) are continuous, f (n ) is piecewise continuous, f , f ′, f ′′, , f ( n )
are absolutely integrable in (−∞ , ∞) and f , f ′, f ′′,  , f ( n −1) → 0 as x → ±∞ , then

                       F ( f ( n ) ( x) ) = (−is ) n F ( s )




                                                                                                                                           13
8. Derivatives of the Transform

                                                                                          dF ( s )
         If F(s) is the Fourier transform of f (x ) then F [ x. f ( x )] = (−i )
                                                                                           ds
Proof:
                                            ∞
                                   1
                       F (s) =              ∫ f ( x )e
                                                         isx
                                                               dx
                                   2π       −∞



                                            ∞

                                            ∫ ds [ f ( x)e ]dx
                     dF ( s )      1          d
         ∴                    =                                 isx

                      ds           2π       −∞



                                            ∞
                                   i
                               =            ∫ [ x. f ( x)]e          dx = iF [ xf ( x)]
                                                               isx

                                   2π       −∞



                     dF ( s)
         ∴ ( −i )            = F [ x. f ( x)]
                      ds


                           [            ]
Extending, we get, F x n . f ( x) = (−i ) n
                                                      d n F (s)
                                                        ds n

DEFINITION

                 ∞
             1
               ∫ f ( x − u ) g (u )du is called the convolution product or simply the convolution
           2π −∞
of the functions f (x) and g (x) and is denoted by f ( x) * g ( x ) .

9. Convolution Theorem

        If F(s) and G(s) are the Fourier transform of f (x ) and g (x) respectively then the
Fourier transform of the convolution of f(x) and g(x) is the product of their Fourier
transforms.
        i.e., F [ f ( x ) * g ( x)] = F ( s ).G ( s)
Proof
                                      ∞
                                  1
         F [ f ( x ) * g ( x )] =     ∫∞ f ( x) * g ( x)e dx
                                                         isx

                                  2π −
                                   1       1 ∞
                                            ∞
                                                                         isx
                               =       ∫∞  2π −∫∞ f ( x − u) g (u )du  e dx
                                   2π −                                
                                   1
                                       ∞
                                                 1 ∞                     
                               =       ∫∞  2π −∫∞ f ( x − u)e dxdu,
                                                                    isx
                                         g (u ) 
                                   2π −                                   
                                      on changing the order of int egration.




                                                                                                     14
∞

                                                    ∫ g (u )[e                         ]
                                           1
                               =                                      ius
                                                                            F ( s) du , by the shifting property.
                                         2π         −∞
                                                                  ∞
                                                         1
                               = F ( s).                          ∫ g (u ).e
                                                                                     ius
                                                                                           du
                                          2π                    −∞

                               = F ( s).G ( s )
Inverting, we get

          F −1 [ F ( s ).G ( s)] = f ( x) * g ( x )
                                     = F −1 { F ( s )} * F −1 { G ( s )}

10. Parseval’s Identity (or) Energy Theorem

        If f (x) is a given function defined in (−∞ , ∞) then it satisfy the identity,
                               ∞                               ∞

                               ∫                                ∫ F ( s)
                                               2                                 2
                                    f ( x) dx =                                      ds
                            −∞                                −∞

where F(s) is the Fourier transform of f (x ) .
Proof:
       We know that F −1 [ F ( s ).G ( s)] = f ( x) * g ( x )

                           ∞                                                               ∞
                     1                                                           1
                           ∫ F (s).G(s)e                                                   ∫ f (t ) g ( x − t )dt
                                                         −isx
                                                                ds =
                2π −∞                                                            2π        −∞
Putting x = 0, we get

                                    ∞                                       ∞

                                    ∫ F (s).G(s)ds = ∫ f (t ) g (−t )dt
                                    −∞                                      −∞
                                                                                                                            ………………..(1)

                               Let                       g ( − t ) = f (t )                                                 .……………….(2)
                                   i.e.,                     g (t ) = f (−t )                                               ………………..(3)
                                                   ∴ G ( s ) = F [ f (− x)] = F ( s )                               by property (9)
                               i.e.,           ∴ G ( s) = F ( s)                                                            ………………..(4)

        Substituting (2) and (4) in (1) we get

                                    ∞                                        ∞

                                    ∫ F (s).F (s) ds =
                                    −∞
                                                                             ∫ f (t ). f (t ) dt
                                                                            −∞




                                                                                                             [ F (s).F (s) = F (s) ]
                                               ∞                             ∞

                                               ∫                             ∫
                                                              2                                 2                                     2
                                                   F ( s) ds =                       f ( x) dx
                                           −∞                               −∞



11. If f (x) and g (x) are given functions of x and FC [ f ( x)] and FC [ g ( x)] are their
Fourier cosine transforms and FS [ f ( x)] and FS [ g ( x)] are their Fourier sine transforms then




                                                                                                                                          15
∞                          ∞                                    ∞

(i)     ∫
        0
            f ( x ) g ( x)dx = ∫ FC [ f ( x)].FC [ g ( x)]ds = ∫ FS [ f ( x )].FS [ g ( x)]ds
                                   0                                    0
        ∞                     ∞                           ∞

        ∫   f ( x) dx = ∫ FC [ f ( x)] ds = ∫ FS [ f ( x )] ds ,
                    2                            2                          2
(ii)
        0                      0                           0

which is Parseval’s identity for Fourier cosine and sine transforms.

Proof:
                ∞                                         ∞
                                                                  2∞                   
(i)             ∫ FC [ f ( x )].FC [ g ( x )]ds = ∫ FC [ f ( x)]    ∫ g ( x) cos sx dx  ds
                0                                 0               π 0                  

                                                           ∞
                                                                     2∞                          
                                                         = ∫ g ( x)     ∫ FC [ f ( x)] cos sx ds  dx,
                                                           0         π 0                         
                                                                      Changing the order of integration
                                                           ∞
                                                         = ∫ f ( x) g ( x)dx
                                                           0

Similarly we can prove the other part of the result.
(ii) Replacing g ( x) = f * ( x) in (i) and noting that FC [ f ( x)] = FC [ f ( x)] and
FS [ f ( x)] = FS [ f ( x)] , we get
                 ∞                          ∞                                   ∞

                 ∫0
                        f ( x ). f ( x).dx = ∫ FC [ f ( x)].FC [ f ( x)] ds = ∫ FS [ f ( x )].FS [ f ( x )] ds
                                             0                                  0


                    ∞                   ∞                           ∞

                    ∫    f ( x) .dx = ∫ FC [ f ( x)] ds = ∫ FS [ f ( x)] ds
                               2                               2                2
i.e.,
                    0                   0                           0



12. If FC [ f ( x)] = FC ( s ) and FS [ f ( x)] = FS ( s ) , then
     d
(i)     { FC ( s)} = − FS { xf ( x)} and
     ds
     d
(ii)    { FS ( s)} = − FC { xf ( x)}.
     ds
Proof:
                                        ∞
                                      2
                                      π∫
                        FC ( s) =         f ( x) cos sx dx
                                        0



                                                 ∞
                            d
                               { FC ( s)} = ∫ f ( x)(− x sin sx)dx
                            ds              0
                                                     ∞
                                            = − ∫ {xf ( x)}sin sx dx
                                                     0



                              = − FS {xf ( x)}
Similarly the result (ii) follows.


                                                                                                                 16
PROBLEMS

                                                   a 2 − x 2
                                                                   x <a
1. Show that the Fourier transform of f ( x ) =                            is
                                                   0
                                                                   x >a>0
                                                ∞
   2  sin as − as cos as                        sin t − t cos t     π
2                         . Hence deduce that ∫                 dt = . Using Parseval’s
   π          s 3
                                               0       t 3
                                                                      4
                          ∞                           2
                      sin t − t cos t      π
identity show that ∫          3        dt = .
                   0                  
                             t               15

Solution:
       We know that
                                   ∞
                              1
         F [ f ( x )] =            ∫ f ( x).e
                                                isx
                                                      dx
                              2π   −∞




                              1                                               
                                   −a              a             ∞
                    =             ∫ f ( x)e dx + ∫ f ( x)e dx + ∫ f ( x )e dx 
                                            isx            isx             isx

                              2π  −∞             −a             a             


                              1                                  
                                       a                                             a
                                                                            1
                    =             0 + ∫ ( a 2 − x 2 )e isx dx + 0 =                ∫ (a
                                                                                            2
                                                                                                − x 2 ).e isx dx
                              2π  − a                                     2π       −a



                                                                                                    a
                              1  2            e isx                e isx   e isx 
                    =             ( a − x 2 )
                                               is          − (−2 x) 2 2  − 2 3 3 
                                                                    i s  i s 
                              2π                                                  − a


                    =
                              1  − 2a isa
                                  2 e +e [−isa  2i
                                                            ]      [
                                                                   
                                                + 3 e −isa − e isa              ]
                              2π  s             s                 

                              1  − 2a             4        
                     =            2 [2 cos as ] + 3 sin as 
                              2π  s              s         

                              4  sin as − as cos as 
                     =
                              2π 
                                         s3         
                                                     

                      2  sin as − as cos as 
             F (s) = 2
                      π         s3         
                                             
                                 2  sin s − s cos s 
       When a = 1, F ( s ) = 2                                                                              ………………..(A)
                                 π       s3        
                                                     
Using inverse Fourier Transform, we get




                                                                                                                      17
∞
                            1          2            1
             f ( x) =
                            2π
                                 .2.
                                       π       ∫s
                                               −∞
                                                    3
                                                        [sin as − as cos as].e −isx ds


                                           ∞
                            1          2 1
                                                { sin as − as cos as}{ cos sx − i sin sx} ds
                                       π −∫ s 3
                    =            .2.
                            2π            ∞



                             ∞
                        2 sin as − as cos as
                        π −∫
             f ( x) =                        cos sx ds
                           ∞      s3

[The second integral is odd and hence its value is zero]

                            ∞
                    4 sin as − as cos as
                  = ∫                    cos sx ds
                    π 0        s3
[since the integrand is an even function of s]

Putting a = 1, we get
                      ∞
                    4 sin s − s cos s
            f ( x) = ∫                cos sx ds
                    π 0     s3

                            ∞
                     4 sin t − t cos t
            f ( x) = ∫                 cos tx dt
                    π 0      t3
Putting x = 0, in the given function we get

                ∞
            4 sin t − t cos t
            π∫
                              dt = f (0) = 1
             0      t3
                ∞
                  sin t − t cos t      π
            ∴   ∫
                0       t 3
                                  dt =
                                       4
                                           ∞                     ∞

                                           ∫                     ∫ F (s)
                                                            2              2
Using Parseval’s identity,                      f ( x) dx =                    ds   [Using (A)]
                                         −∞                      −∞
                                                        2
                   2                   
             ∞  2.   (sin s − s cos s)       1
                    π
             ∫ 
            − ∞          s3
                                         ds = ∫ (1 − x 2 ) 2 dx
                                              −1
                                       
                                       
                        ∞                               2        1
                    8  sin s − s cos s 
                     ∫∞                 ds = 2 ∫ (1 − x ) dx
                                                         2 2

                    π−       s 3
                                                0
                        ∞                               2
                    16  sin s − s cos s           8
                      ∫
                    π 0       s 3        ds = 2.
                                                  15




                                                                                                  18
∞                           2
                              sin s − s cos s       π
                i.e.,       ∫
                            0       s 3        ds =
                                                     15
                             ∞                       2
                               sin t − t cos t       π
                i.e.,        ∫ t3
                             0
                                                 dt =
                                                      15

2. Find the Fourier Transform of f (x) if
                  1 − x , x < 1
                  
         f ( x) = 
                  0,
                          x >1
                                ∞              4
                           sin t          π
Hence deduce that ∫               dt =
                         0
                              t            3
Solution:
We know that
                                 ∞
                            1
          F [ f ( x )] =         ∫∞ f ( x).e dx
                                            isx

                            2π −

                                         1

                                         ∫ (1 − x ).e
                                    1
                            =                           isx
                                                              dx
                                    2π   −1



Since x > 1, f ( x) = 0, i.e., in − ∞ < x < −1, and 1 < x < ∞, f ( x ) = 0.
                                          1

                                          ∫ [1 − x ] [ cos sx + i sin sx] dx
                                    1
                             =
                                    2π    −1



                                          1

                          ∫ [1 − x ] cos sx dx
                                    1
                             =
                       2π −1
The second integral becomes zero since it is an odd function.
                                          1
                                    2
                             =
                                    2π
                                          ∫ (1 − x) cos sx dx
                                          0

                                             [ [1 − x ] cos x is an even function]
                                                                               1
                               2                  sin sx    − cos sx 
                             =           (1 − x ) s  − (−1) s 2 
                               π                                      0


                                    2  − cos s 1 
                             =                + 2
                                    π  s2      s 


                                    2 1
        i.e.,           F ( s) =     . (1 − cos s )
                                    π s2


                                                                                      19
Using Parseval’s identity

            ∞                             ∞

            ∫                             ∫ F ( s)
                             2                         2
                 f ( x) dx =                               ds
            −∞                            −∞



            1                                  ∞
               [1 − x ] dx = 2 ∫ (1 − cos s) ds
                                            2

            ∫
                             2

            −1
                             π −∞     s4
                 1                                 ∞
                                           2 (1 − cos s) 2
            2 ∫ [1 − x ] dx =
                                           π −∫
                                 2
                                                           ds
                 0                            ∞   s4

                                               ∞
                                     2 2 (1 − cos s ) 2
                                     3 π −∫
                                      =                 ds
                                          ∞   s4


             put              s = 2t                    when s = ∞, t = ∞
                             ds = 2dt                   when s = −∞, t = −∞

                                     π ∞ (1 − cos 2t ) 2
                                     3 −∫
                                      =                  .2 dt
                                        ∞    16t 4


                                      π ∞ (1 − cos 2t ) 2
                                      3 −∫
                                       =                  dt
                                         ∞     8t 4

                                           ∞
                                      π      (1 − cos 2t ) 2
                                        = 2∫                 dt
                                      3    0      8t 4


                                      π ∞ sin 4 t
                                      3 ∫ t4
                                       =          dt
                                         0



                     ∞                4
                        sin t   π
    i.e.,            ∫  t  dt = 3
                     0        

                         ∞
                                         dx
3. Evaluate              ∫ (x
                         0
                                 2
                                     + a )( x 2 + b 2 )
                                          2             using transforms.


Solution:
                                                                                       2     a
      We know that the Fourier cosine transform of f ( x) = e − ax is                    . 2    .
                                                                                       π s + a2
                                                                              2     b
Similarly the Fourier cosine transform of f ( x) = e − ax is                    . 2    .
                                                                              π s + b2


                                                                                                    20
∞                                       ∞

We know that           ∫ FC [ f ( x)].FC [ g ( x)] ds = ∫ f ( x).g ( x) dx
                       0                                       0
                   ∞                                                ∞
                           2     a     2   b
    i.e.,          ∫
                   0
                             . 2     . . 2
                           π s +a π s +b
                                   2         2
                                               . ds = ∫ e − ax .e −bx dx
                                                      0



                                   ∞                                 ∞
                               2             ab
                                 ∫ (s 2 + a 2 )(s 2 + b 2 ) ds = ∫ e
                                                                     −( a +b ) x
    i.e.,                                                                        ds
                               π 0                               0



                                                                    ∞
                                                      e −( a + b ) x            1       1
                                                    =                 = 0−           =
                                                      − ( a + b)  0        − ( a + b) a + b

                   ∞
                                   dx             π
    i.e.,          ∫ (x
                   0
                           2      2    2   2
                                             =
                               + a )( x + b ) 2ab( a + b)

4. Find the Fourier transform of e − a x and hence deduce that
    ∞
       cos xt    π −a x
(i) ∫ 2 2 dt = e
    0 a +t
                2a

      [
(ii) F xe
            −a x
                   ]=i         2     2as
                               π (s + a 2 ) 2
                                   2


Solution:
                                       ∞
                                  1
          F [ f ( x )] =               ∫ f ( x).e
                                                    isx
                                                          dx
                                  2π   −∞




                                 1                                        
                                       0                  ∞
                           =        ∫   f ( x)e isx dx + ∫ f ( x)e isx dx 
                                 2π  −∞                  0                


                                         e − ax
                                                         if 0 ≤ x < ∞
          Here                 f ( x ) =  ax
                                         e
                                                         if − ∞ < x < 0


                                 1  ax isx                          
                                       0           ∞
                           =        ∫   e. e dx + ∫ e −ax .e isx dx 
                                 2π  −∞           0                 


                                 1  ( a +is ) x                          
                                       0              ∞
                           =         ∫ e.       dx + ∫ e −( a −is ) x dx 
                                 2π  −∞              0                   




                                                                                                21
1  e ( a +is ) x     e −( a −is ) x  
                                            0                   ∞

                  =                      +                  
                       2π  (a + is)  −∞  − (a − is )  0 
                                                                 

                       1  1           1 
                  =        a + is + a − is 
                       2π                  


        Fe[   −a x
                      ]=    2    a
                            π s + a2
                               2



Using inversion formula, we get

                                    ∞
                                1        2     a
              f ( x) =
                            2π
                                    ∫
                                    −∞
                                           . 2
                                         π s +a  2
                                                   e −isx ds


                                ∞
                           a cos sx − i sin sx
                           π −∫
                       =                       ds
                              ∞  s2 + a2

                                ∞
                           a      cos x
                       =
                           π−∫∞ s 2 + a 2 ds

              ∞
                  cos sx       π           π −a x
              ∫s
              0
                   2
                     +a 2
                          dx =
                               2a
                                  f ( x) =
                                           2a
                                              .e                       (or )


              ∞
                  cos tx       π −a x
              ∫s
              0
                  2
                    +a  2
                          dt =
                               2a
                                  .e


Putting a = 1, we get,


                     Fe[ ]=−x           2 1
                                         .
                                        π s2 +1

                            ∞                                      ∞
                              cos sx       π −x                      cos tx     π −x
                     and    ∫ s 2 + 1 ds = 2 e
                            0
                                                           (or )   ∫t
                                                                   0
                                                                      2
                                                                        +1
                                                                            dt = e
                                                                                2


FINITE FOURIER TRANSFORMS

       If f (x) is a function defined in the interval (0 , l) then the finite Fourier sine
transform of f (x) in 0 <x < l is defined as




                                                                                             22
nπx
                                        l
                  FS [ f ( x)] = ∫ f ( x). sin           dx         where ‘n’ is an integer
                                        0
                                                      l

       The inverse finite Fourier sine transform of FS [ f (x )] is f (x) and is given by
                       2 ∞                  nπx
               f ( x) = ∑ FS [ f ( x )] sin
                       l n =1                l

       The finite Fourier cosine transform of f (x ) in 0 < x < l is defined as
                                                       nπx
                                                l
                   FC [ f ( x)] = ∫ f ( x). cos            dx         where ‘n’ is an integer
                                                0
                                                        l

       The inverse finite Fourier cosine transform of FC [ f (x)] is f (x) and is given by
                       1        2 ∞                  nπx
               f ( x) = FC (0) + ∑ FC [ f ( x )] cos
                       l        l n =1                l

PROBLEMS

1. Find the finite Fourier sine and cosine transforms of f ( x) = x 2 in 0 < x < l.

Solution:
       The finite Fourier sine transform is

                                                    nπx
                                    l
                 FS [ f ( x)] = ∫ f ( x). sin           dx
                                    0
                                                     l
Here      f ( x) = x   2




                              [ ]                   nπx
                                            l
                           FS x 2 = ∫ x 2 . sin         dx
                                            0
                                                     l

                                                                                                l
                                              nπx              nπx       nπx 
                                        − cos           − sin        cos     
                                    = x 2      l  − 2 x         l  + 2    l 
                                           nπ           n 2π 2   n 3π 3 
                                                                            
                                             l               l2       l 3  0  


                                            − l3          2l 3         2l 3
                                    =            cos nπ + 3 3 cos nπ − 3 3
                                            nπ           nπ           nπ


                                    =
                                            l3
                                            nπ
                                                           2l 3
                                                                [
                                               (−1) n +1 + 3 3 (−1) n − 1
                                                          nπ
                                                                            ]
The finite Fourier cosine transform is



                                                                                                    23
nπx
                                        l
                     FC [ f ( x)] = ∫ f ( x). cos       dx
                                        0
                                                     l
Here        f ( x) = x   2




                             [ ]                 nπx
                                        l
                         FC x 2 = ∫ x 2 . cos        dx
                                        0
                                                  l
                                                                              l
                             nπx              nπx          nπx 
                         sin           − cos        − sin      
                     = x 2    l  − 2 x         l  + 2       l 
                         nπ            n 2π 2   n 3π 3 
                         l                                    
                                            l2           l3      0


                      2l 3
                 =          cos nπ
                     n 2π 2

                     2l 3
                 =        (−1) n
                     nπ
                      2 2



2. Find the finite Fourier sine and cosine transforms of f ( x ) = x in (0 , π ) .

Solution:
       The finite Fourier sine transform of f ( x) = x in (0 , π ) is
                                   π
                 FS [ f ( x)] = ∫ f ( x ). sin nx dx
                                   0

Here        f ( x) = x in (0 , π )
                                   π                                              π
                                                    − cos nx   − sin nx 
                      FS [ x ] = ∫ x. sin nx dx =  x          − 1  2    
                                 0                     n       n         0

                                       π                      π
                              =−         cos nπ = (−1) n +1 .
                                       n                      n

The finite Fourier cosine transform of f ( x) = x in (0 , π ) is
                                   π
                 FC [ f ( x)] = ∫ f ( x). cos nx dx
                                   0

Here        f ( x) = x in (0 , π )
                                   π                                         π
                                                    sin nx   − cos nx 
                      FC [ x ] = ∫ x. cos nx dx =  x        − 1  2    
                                 0                  n   n              0


                              =
                                   1
                                   n 2
                                                1   1
                                                             [
                                       cos nπ − 2 = 2 (−1) n − 1
                                               n   n
                                                                  ]


                                                                                      24
2π (−1) p−1
3. Find f (x) if its finite sine transform is given by                                          , where p is positive
                                                                                        p3
integer and 0 < x < π .

Solution:
       We know that the inverse Fourier sine transform is given by

                                       ∞
                               2
                    f ( x) =        ∑        FS [ f ( x )] sin px                                          ………………..(1)
                               π     p =1

                              2π (−1) p−1
Here     FS [ f (x )] =                                                                                    ………………..(2)
                                  p3

Substituting (2) in (1), we get
                   2 ∞ 2π (−1) p −1
           f ( x) = ∑               sin px
                   π p =1    p3

                         ∞
                             (−1) p −1
                    = 4∑               sin px
                        p =1   p3

                    2 pπ 
               cos         
                    3  find FC [ f ( p )] if 0 < x <1.
                                −1
4. If
      f ( p) =
                (2 p + 1) 2

Solution:
                                                                           ∞
                                                                                                     nπx
       We know that FC
                                     −1
                                           [ f ( p)] = 1 FC (0) + 2 ∑             FC [ f ( x)] cos
                                                         l             l   n =1                       l
                       2 pπ 
                  cos         
Here
         f ( p) =      3 
                   (2 p + 1) 2

Let    FC [ f ( x)] = f ( p)

                                                  ∞
                                                                       nπx
 ∴ FC
         −1
              [ f ( p)] = 1    f C ( 0) +
                                              2
                                                  ∑      f ( p ) cos                      [ l = 1]
                          l                   l   n =1                  l


                                                 2 pπ 
                                   ∞
                                            cos         
                      = 1 + 2∑                   3  . cos nπx
                                   n =1      (2 p + 1) 2




                                                                                                                        25
UNIT-4
                                                                         PART A

1. State the Fourier integral theorem.
Ans:
       If f (x) is a given function defined in (-l , l) and satisfies Dirichlet’s conditions, then
                                     ∞ ∞
                                 1
                  f ( x) =
                                 π   ∫ ∫ f (t ) cos λ (t − x) dt dλ
                                         0 −∞



2. State the convolution theorem of the Fourier transform.
Ans:
        If F(s) and G(s) are the Fourier transform of f (x ) and g (x) respectively then the
Fourier transform of the convolution of f(x) and g(x) is the product of their Fourier
transforms.
        i.e., F [ f ( x ) * g ( x)] = F ( s ).G ( s)

3. Write the Fourier transform pair.
Ans:
F [ f (x)] and F −1 [ F ( S )] are Fourier transform pairs.

4. Find the Fourier sine transform of f ( x) = e − ax (a > 0).
Ans:
                      ∞
                    2
     FS [ f ( x)] =
                    π ∫
                        f ( x ). sin sx dx
                      0

                      2
                           ∞
                                                                   2 s                            ∞ −ax              b 
                           ∫ e . sin sx dx =                                                        ∫ e sin bx dx = 2
                              − ax
                 =                                                                                                          
                      π      0
                                                                   π  s2 + a2 
                                                                                                  0               a + b2 
                      2 s 
                 =
                      π  s2 + a2 
                                 

5. If the Fourier transform of f (x ) is F(s) then prove that . F [ f ( x − a )] = e isa F ( s)
Ans:
                                ∞
                             1
           F [ f ( x − a)] =     ∫∞ f ( x − a).e dx
                                                isx

                             2π −
Put        x-a = y
           dx = dy
When x = −∞, y = −∞ and x = ∞, y = ∞
                                            ∞                                             ∞
                                   1                                              e ias
         F [ f ( x − a)] =                  ∫ f ( y).e
                                                          is ( y + a )
                                                                         . dy =           ∫ f ( y).e
                                                                                                       isy
                                                                                                             .dy
                                   2π       −∞                                     2π     −∞



                                            ∞
                                 e ias
                       =                    ∫ f ( x).e         .dx = e isa F ( s )
                                                         isx

                                  2π       −∞


6. State the Fourier transforms of the derivatives of a function.


                                                                                                                                26
Chapter 4 (maths 3)
Chapter 4 (maths 3)
Chapter 4 (maths 3)
Chapter 4 (maths 3)
Chapter 4 (maths 3)

Contenu connexe

Tendances

Linear differential equation
Linear differential equationLinear differential equation
Linear differential equationPratik Sudra
 
Discrete fourier transform
Discrete fourier transformDiscrete fourier transform
Discrete fourier transformMOHAMMAD AKRAM
 
Ordinary differential equations
Ordinary differential equationsOrdinary differential equations
Ordinary differential equationsAhmed Haider
 
inverse z-transform ppt
inverse z-transform pptinverse z-transform ppt
inverse z-transform pptmihir jain
 
Dsp U Lec05 The Z Transform
Dsp U   Lec05 The Z TransformDsp U   Lec05 The Z Transform
Dsp U Lec05 The Z Transformtaha25
 
Ode powerpoint presentation1
Ode powerpoint presentation1Ode powerpoint presentation1
Ode powerpoint presentation1Pokkarn Narkhede
 
Application of analytic function
Application of analytic functionApplication of analytic function
Application of analytic functionDr. Nirav Vyas
 
Laplace Transformation & Its Application
Laplace Transformation & Its ApplicationLaplace Transformation & Its Application
Laplace Transformation & Its ApplicationChandra Kundu
 
Chapter2 - Linear Time-Invariant System
Chapter2 - Linear Time-Invariant SystemChapter2 - Linear Time-Invariant System
Chapter2 - Linear Time-Invariant SystemAttaporn Ninsuwan
 
Gauss Divergence Therom
Gauss Divergence TheromGauss Divergence Therom
Gauss Divergence TheromVC Infotech
 
Applications of differential equations
Applications of differential equationsApplications of differential equations
Applications of differential equationsSohag Babu
 
APPLICATION OF PARTIAL DIFFERENTIATION
APPLICATION OF PARTIAL DIFFERENTIATIONAPPLICATION OF PARTIAL DIFFERENTIATION
APPLICATION OF PARTIAL DIFFERENTIATIONDhrupal Patel
 
Fourier series Introduction
Fourier series IntroductionFourier series Introduction
Fourier series IntroductionRizwan Kazi
 

Tendances (20)

Linear differential equation
Linear differential equationLinear differential equation
Linear differential equation
 
Discrete fourier transform
Discrete fourier transformDiscrete fourier transform
Discrete fourier transform
 
Ordinary differential equations
Ordinary differential equationsOrdinary differential equations
Ordinary differential equations
 
inverse z-transform ppt
inverse z-transform pptinverse z-transform ppt
inverse z-transform ppt
 
Dsp U Lec05 The Z Transform
Dsp U   Lec05 The Z TransformDsp U   Lec05 The Z Transform
Dsp U Lec05 The Z Transform
 
Ode powerpoint presentation1
Ode powerpoint presentation1Ode powerpoint presentation1
Ode powerpoint presentation1
 
Fourier series
Fourier seriesFourier series
Fourier series
 
Application of analytic function
Application of analytic functionApplication of analytic function
Application of analytic function
 
Gamma function
Gamma functionGamma function
Gamma function
 
Laplace Transformation & Its Application
Laplace Transformation & Its ApplicationLaplace Transformation & Its Application
Laplace Transformation & Its Application
 
Z transform
Z transformZ transform
Z transform
 
Stoke’s theorem
Stoke’s theoremStoke’s theorem
Stoke’s theorem
 
Properties of dft
Properties of dftProperties of dft
Properties of dft
 
Calculus of variations
Calculus of variationsCalculus of variations
Calculus of variations
 
Chapter2 - Linear Time-Invariant System
Chapter2 - Linear Time-Invariant SystemChapter2 - Linear Time-Invariant System
Chapter2 - Linear Time-Invariant System
 
Secant Method
Secant MethodSecant Method
Secant Method
 
Gauss Divergence Therom
Gauss Divergence TheromGauss Divergence Therom
Gauss Divergence Therom
 
Applications of differential equations
Applications of differential equationsApplications of differential equations
Applications of differential equations
 
APPLICATION OF PARTIAL DIFFERENTIATION
APPLICATION OF PARTIAL DIFFERENTIATIONAPPLICATION OF PARTIAL DIFFERENTIATION
APPLICATION OF PARTIAL DIFFERENTIATION
 
Fourier series Introduction
Fourier series IntroductionFourier series Introduction
Fourier series Introduction
 

En vedette

N.P. BALI ENGINEERING MATH REAL
N.P. BALI ENGINEERING MATH REALN.P. BALI ENGINEERING MATH REAL
N.P. BALI ENGINEERING MATH REALchucky oz
 
Numerical Solution of Nth - Order Fuzzy Initial Value Problems by Fourth Orde...
Numerical Solution of Nth - Order Fuzzy Initial Value Problems by Fourth Orde...Numerical Solution of Nth - Order Fuzzy Initial Value Problems by Fourth Orde...
Numerical Solution of Nth - Order Fuzzy Initial Value Problems by Fourth Orde...IOSR Journals
 
Free Ebooks Download ! Edhole
Free Ebooks Download ! EdholeFree Ebooks Download ! Edhole
Free Ebooks Download ! EdholeEdhole.com
 
Free Download Powerpoint Slides
Free Download Powerpoint SlidesFree Download Powerpoint Slides
Free Download Powerpoint SlidesGeorge
 

En vedette (9)

Chapter 2 (maths 3)
Chapter 2 (maths 3)Chapter 2 (maths 3)
Chapter 2 (maths 3)
 
Chapter 3 (maths 3)
Chapter 3 (maths 3)Chapter 3 (maths 3)
Chapter 3 (maths 3)
 
Chapter 1 (maths 3)
Chapter 1 (maths 3)Chapter 1 (maths 3)
Chapter 1 (maths 3)
 
Chapter 5 (maths 3)
Chapter 5 (maths 3)Chapter 5 (maths 3)
Chapter 5 (maths 3)
 
mathematics formulas
mathematics formulasmathematics formulas
mathematics formulas
 
N.P. BALI ENGINEERING MATH REAL
N.P. BALI ENGINEERING MATH REALN.P. BALI ENGINEERING MATH REAL
N.P. BALI ENGINEERING MATH REAL
 
Numerical Solution of Nth - Order Fuzzy Initial Value Problems by Fourth Orde...
Numerical Solution of Nth - Order Fuzzy Initial Value Problems by Fourth Orde...Numerical Solution of Nth - Order Fuzzy Initial Value Problems by Fourth Orde...
Numerical Solution of Nth - Order Fuzzy Initial Value Problems by Fourth Orde...
 
Free Ebooks Download ! Edhole
Free Ebooks Download ! EdholeFree Ebooks Download ! Edhole
Free Ebooks Download ! Edhole
 
Free Download Powerpoint Slides
Free Download Powerpoint SlidesFree Download Powerpoint Slides
Free Download Powerpoint Slides
 

Similaire à Chapter 4 (maths 3)

Lesson 14: Derivatives of Logarithmic and Exponential Functions (slides)
Lesson 14: Derivatives of Logarithmic and Exponential Functions (slides)Lesson 14: Derivatives of Logarithmic and Exponential Functions (slides)
Lesson 14: Derivatives of Logarithmic and Exponential Functions (slides)Matthew Leingang
 
Lesson 14: Derivatives of Logarithmic and Exponential Functions (slides)
Lesson 14: Derivatives of Logarithmic and Exponential Functions (slides)Lesson 14: Derivatives of Logarithmic and Exponential Functions (slides)
Lesson 14: Derivatives of Logarithmic and Exponential Functions (slides)Mel Anthony Pepito
 
SOME THOUGHTS ON DIVERGENT SERIES
SOME THOUGHTS ON DIVERGENT SERIESSOME THOUGHTS ON DIVERGENT SERIES
SOME THOUGHTS ON DIVERGENT SERIESgenius98
 
some thoughts on divergent series
some thoughts on divergent seriessome thoughts on divergent series
some thoughts on divergent seriesgenius98
 
Varian, microeconomic analysis, solution book
Varian, microeconomic analysis, solution bookVarian, microeconomic analysis, solution book
Varian, microeconomic analysis, solution bookJosé Antonio PAYANO YALE
 
Lesson 28: The Fundamental Theorem of Calculus
Lesson 28: The Fundamental Theorem of CalculusLesson 28: The Fundamental Theorem of Calculus
Lesson 28: The Fundamental Theorem of CalculusMatthew Leingang
 
Lesson 28: The Fundamental Theorem of Calculus
Lesson 28: The Fundamental Theorem of CalculusLesson 28: The Fundamental Theorem of Calculus
Lesson 28: The Fundamental Theorem of CalculusMatthew Leingang
 
Lattices and codes
Lattices and codesLattices and codes
Lattices and codesSpringer
 
Introduction to inverse problems
Introduction to inverse problemsIntroduction to inverse problems
Introduction to inverse problemsDelta Pi Systems
 
Lesson 26: The Fundamental Theorem of Calculus (slides)
Lesson 26: The Fundamental Theorem of Calculus (slides)Lesson 26: The Fundamental Theorem of Calculus (slides)
Lesson 26: The Fundamental Theorem of Calculus (slides)Matthew Leingang
 
Lesson 26: The Fundamental Theorem of Calculus (slides)
Lesson 26: The Fundamental Theorem of Calculus (slides)Lesson 26: The Fundamental Theorem of Calculus (slides)
Lesson 26: The Fundamental Theorem of Calculus (slides)Mel Anthony Pepito
 
Lesson 26: The Fundamental Theorem of Calculus (slides)
Lesson 26: The Fundamental Theorem of Calculus (slides)Lesson 26: The Fundamental Theorem of Calculus (slides)
Lesson 26: The Fundamental Theorem of Calculus (slides)Matthew Leingang
 
IJCER (www.ijceronline.com) International Journal of computational Engineerin...
IJCER (www.ijceronline.com) International Journal of computational Engineerin...IJCER (www.ijceronline.com) International Journal of computational Engineerin...
IJCER (www.ijceronline.com) International Journal of computational Engineerin...ijceronline
 

Similaire à Chapter 4 (maths 3) (20)

Fourier series
Fourier seriesFourier series
Fourier series
 
Lesson 14: Derivatives of Logarithmic and Exponential Functions (slides)
Lesson 14: Derivatives of Logarithmic and Exponential Functions (slides)Lesson 14: Derivatives of Logarithmic and Exponential Functions (slides)
Lesson 14: Derivatives of Logarithmic and Exponential Functions (slides)
 
Lesson 14: Derivatives of Logarithmic and Exponential Functions (slides)
Lesson 14: Derivatives of Logarithmic and Exponential Functions (slides)Lesson 14: Derivatives of Logarithmic and Exponential Functions (slides)
Lesson 14: Derivatives of Logarithmic and Exponential Functions (slides)
 
SOME THOUGHTS ON DIVERGENT SERIES
SOME THOUGHTS ON DIVERGENT SERIESSOME THOUGHTS ON DIVERGENT SERIES
SOME THOUGHTS ON DIVERGENT SERIES
 
some thoughts on divergent series
some thoughts on divergent seriessome thoughts on divergent series
some thoughts on divergent series
 
Ism et chapter_3
Ism et chapter_3Ism et chapter_3
Ism et chapter_3
 
Ism et chapter_3
Ism et chapter_3Ism et chapter_3
Ism et chapter_3
 
Ism et chapter_3
Ism et chapter_3Ism et chapter_3
Ism et chapter_3
 
Funcion gamma
Funcion gammaFuncion gamma
Funcion gamma
 
Taylor problem
Taylor problemTaylor problem
Taylor problem
 
Varian, microeconomic analysis, solution book
Varian, microeconomic analysis, solution bookVarian, microeconomic analysis, solution book
Varian, microeconomic analysis, solution book
 
Fourier series
Fourier seriesFourier series
Fourier series
 
Lesson 28: The Fundamental Theorem of Calculus
Lesson 28: The Fundamental Theorem of CalculusLesson 28: The Fundamental Theorem of Calculus
Lesson 28: The Fundamental Theorem of Calculus
 
Lesson 28: The Fundamental Theorem of Calculus
Lesson 28: The Fundamental Theorem of CalculusLesson 28: The Fundamental Theorem of Calculus
Lesson 28: The Fundamental Theorem of Calculus
 
Lattices and codes
Lattices and codesLattices and codes
Lattices and codes
 
Introduction to inverse problems
Introduction to inverse problemsIntroduction to inverse problems
Introduction to inverse problems
 
Lesson 26: The Fundamental Theorem of Calculus (slides)
Lesson 26: The Fundamental Theorem of Calculus (slides)Lesson 26: The Fundamental Theorem of Calculus (slides)
Lesson 26: The Fundamental Theorem of Calculus (slides)
 
Lesson 26: The Fundamental Theorem of Calculus (slides)
Lesson 26: The Fundamental Theorem of Calculus (slides)Lesson 26: The Fundamental Theorem of Calculus (slides)
Lesson 26: The Fundamental Theorem of Calculus (slides)
 
Lesson 26: The Fundamental Theorem of Calculus (slides)
Lesson 26: The Fundamental Theorem of Calculus (slides)Lesson 26: The Fundamental Theorem of Calculus (slides)
Lesson 26: The Fundamental Theorem of Calculus (slides)
 
IJCER (www.ijceronline.com) International Journal of computational Engineerin...
IJCER (www.ijceronline.com) International Journal of computational Engineerin...IJCER (www.ijceronline.com) International Journal of computational Engineerin...
IJCER (www.ijceronline.com) International Journal of computational Engineerin...
 

Dernier

FSB Advising Checklist - Orientation 2024
FSB Advising Checklist - Orientation 2024FSB Advising Checklist - Orientation 2024
FSB Advising Checklist - Orientation 2024Elizabeth Walsh
 
Unit 3 Emotional Intelligence and Spiritual Intelligence.pdf
Unit 3 Emotional Intelligence and Spiritual Intelligence.pdfUnit 3 Emotional Intelligence and Spiritual Intelligence.pdf
Unit 3 Emotional Intelligence and Spiritual Intelligence.pdfDr Vijay Vishwakarma
 
Kodo Millet PPT made by Ghanshyam bairwa college of Agriculture kumher bhara...
Kodo Millet  PPT made by Ghanshyam bairwa college of Agriculture kumher bhara...Kodo Millet  PPT made by Ghanshyam bairwa college of Agriculture kumher bhara...
Kodo Millet PPT made by Ghanshyam bairwa college of Agriculture kumher bhara...pradhanghanshyam7136
 
Unit-V; Pricing (Pharma Marketing Management).pptx
Unit-V; Pricing (Pharma Marketing Management).pptxUnit-V; Pricing (Pharma Marketing Management).pptx
Unit-V; Pricing (Pharma Marketing Management).pptxVishalSingh1417
 
Making communications land - Are they received and understood as intended? we...
Making communications land - Are they received and understood as intended? we...Making communications land - Are they received and understood as intended? we...
Making communications land - Are they received and understood as intended? we...Association for Project Management
 
The basics of sentences session 3pptx.pptx
The basics of sentences session 3pptx.pptxThe basics of sentences session 3pptx.pptx
The basics of sentences session 3pptx.pptxheathfieldcps1
 
On National Teacher Day, meet the 2024-25 Kenan Fellows
On National Teacher Day, meet the 2024-25 Kenan FellowsOn National Teacher Day, meet the 2024-25 Kenan Fellows
On National Teacher Day, meet the 2024-25 Kenan FellowsMebane Rash
 
How to Give a Domain for a Field in Odoo 17
How to Give a Domain for a Field in Odoo 17How to Give a Domain for a Field in Odoo 17
How to Give a Domain for a Field in Odoo 17Celine George
 
ICT role in 21st century education and it's challenges.
ICT role in 21st century education and it's challenges.ICT role in 21st century education and it's challenges.
ICT role in 21st century education and it's challenges.MaryamAhmad92
 
80 ĐỀ THI THỬ TUYỂN SINH TIẾNG ANH VÀO 10 SỞ GD – ĐT THÀNH PHỐ HỒ CHÍ MINH NĂ...
80 ĐỀ THI THỬ TUYỂN SINH TIẾNG ANH VÀO 10 SỞ GD – ĐT THÀNH PHỐ HỒ CHÍ MINH NĂ...80 ĐỀ THI THỬ TUYỂN SINH TIẾNG ANH VÀO 10 SỞ GD – ĐT THÀNH PHỐ HỒ CHÍ MINH NĂ...
80 ĐỀ THI THỬ TUYỂN SINH TIẾNG ANH VÀO 10 SỞ GD – ĐT THÀNH PHỐ HỒ CHÍ MINH NĂ...Nguyen Thanh Tu Collection
 
Single or Multiple melodic lines structure
Single or Multiple melodic lines structureSingle or Multiple melodic lines structure
Single or Multiple melodic lines structuredhanjurrannsibayan2
 
Food safety_Challenges food safety laboratories_.pdf
Food safety_Challenges food safety laboratories_.pdfFood safety_Challenges food safety laboratories_.pdf
Food safety_Challenges food safety laboratories_.pdfSherif Taha
 
Jamworks pilot and AI at Jisc (20/03/2024)
Jamworks pilot and AI at Jisc (20/03/2024)Jamworks pilot and AI at Jisc (20/03/2024)
Jamworks pilot and AI at Jisc (20/03/2024)Jisc
 
2024-NATIONAL-LEARNING-CAMP-AND-OTHER.pptx
2024-NATIONAL-LEARNING-CAMP-AND-OTHER.pptx2024-NATIONAL-LEARNING-CAMP-AND-OTHER.pptx
2024-NATIONAL-LEARNING-CAMP-AND-OTHER.pptxMaritesTamaniVerdade
 
Holdier Curriculum Vitae (April 2024).pdf
Holdier Curriculum Vitae (April 2024).pdfHoldier Curriculum Vitae (April 2024).pdf
Holdier Curriculum Vitae (April 2024).pdfagholdier
 
This PowerPoint helps students to consider the concept of infinity.
This PowerPoint helps students to consider the concept of infinity.This PowerPoint helps students to consider the concept of infinity.
This PowerPoint helps students to consider the concept of infinity.christianmathematics
 
SOC 101 Demonstration of Learning Presentation
SOC 101 Demonstration of Learning PresentationSOC 101 Demonstration of Learning Presentation
SOC 101 Demonstration of Learning Presentationcamerronhm
 
Basic Civil Engineering first year Notes- Chapter 4 Building.pptx
Basic Civil Engineering first year Notes- Chapter 4 Building.pptxBasic Civil Engineering first year Notes- Chapter 4 Building.pptx
Basic Civil Engineering first year Notes- Chapter 4 Building.pptxDenish Jangid
 
Sociology 101 Demonstration of Learning Exhibit
Sociology 101 Demonstration of Learning ExhibitSociology 101 Demonstration of Learning Exhibit
Sociology 101 Demonstration of Learning Exhibitjbellavia9
 
How to Create and Manage Wizard in Odoo 17
How to Create and Manage Wizard in Odoo 17How to Create and Manage Wizard in Odoo 17
How to Create and Manage Wizard in Odoo 17Celine George
 

Dernier (20)

FSB Advising Checklist - Orientation 2024
FSB Advising Checklist - Orientation 2024FSB Advising Checklist - Orientation 2024
FSB Advising Checklist - Orientation 2024
 
Unit 3 Emotional Intelligence and Spiritual Intelligence.pdf
Unit 3 Emotional Intelligence and Spiritual Intelligence.pdfUnit 3 Emotional Intelligence and Spiritual Intelligence.pdf
Unit 3 Emotional Intelligence and Spiritual Intelligence.pdf
 
Kodo Millet PPT made by Ghanshyam bairwa college of Agriculture kumher bhara...
Kodo Millet  PPT made by Ghanshyam bairwa college of Agriculture kumher bhara...Kodo Millet  PPT made by Ghanshyam bairwa college of Agriculture kumher bhara...
Kodo Millet PPT made by Ghanshyam bairwa college of Agriculture kumher bhara...
 
Unit-V; Pricing (Pharma Marketing Management).pptx
Unit-V; Pricing (Pharma Marketing Management).pptxUnit-V; Pricing (Pharma Marketing Management).pptx
Unit-V; Pricing (Pharma Marketing Management).pptx
 
Making communications land - Are they received and understood as intended? we...
Making communications land - Are they received and understood as intended? we...Making communications land - Are they received and understood as intended? we...
Making communications land - Are they received and understood as intended? we...
 
The basics of sentences session 3pptx.pptx
The basics of sentences session 3pptx.pptxThe basics of sentences session 3pptx.pptx
The basics of sentences session 3pptx.pptx
 
On National Teacher Day, meet the 2024-25 Kenan Fellows
On National Teacher Day, meet the 2024-25 Kenan FellowsOn National Teacher Day, meet the 2024-25 Kenan Fellows
On National Teacher Day, meet the 2024-25 Kenan Fellows
 
How to Give a Domain for a Field in Odoo 17
How to Give a Domain for a Field in Odoo 17How to Give a Domain for a Field in Odoo 17
How to Give a Domain for a Field in Odoo 17
 
ICT role in 21st century education and it's challenges.
ICT role in 21st century education and it's challenges.ICT role in 21st century education and it's challenges.
ICT role in 21st century education and it's challenges.
 
80 ĐỀ THI THỬ TUYỂN SINH TIẾNG ANH VÀO 10 SỞ GD – ĐT THÀNH PHỐ HỒ CHÍ MINH NĂ...
80 ĐỀ THI THỬ TUYỂN SINH TIẾNG ANH VÀO 10 SỞ GD – ĐT THÀNH PHỐ HỒ CHÍ MINH NĂ...80 ĐỀ THI THỬ TUYỂN SINH TIẾNG ANH VÀO 10 SỞ GD – ĐT THÀNH PHỐ HỒ CHÍ MINH NĂ...
80 ĐỀ THI THỬ TUYỂN SINH TIẾNG ANH VÀO 10 SỞ GD – ĐT THÀNH PHỐ HỒ CHÍ MINH NĂ...
 
Single or Multiple melodic lines structure
Single or Multiple melodic lines structureSingle or Multiple melodic lines structure
Single or Multiple melodic lines structure
 
Food safety_Challenges food safety laboratories_.pdf
Food safety_Challenges food safety laboratories_.pdfFood safety_Challenges food safety laboratories_.pdf
Food safety_Challenges food safety laboratories_.pdf
 
Jamworks pilot and AI at Jisc (20/03/2024)
Jamworks pilot and AI at Jisc (20/03/2024)Jamworks pilot and AI at Jisc (20/03/2024)
Jamworks pilot and AI at Jisc (20/03/2024)
 
2024-NATIONAL-LEARNING-CAMP-AND-OTHER.pptx
2024-NATIONAL-LEARNING-CAMP-AND-OTHER.pptx2024-NATIONAL-LEARNING-CAMP-AND-OTHER.pptx
2024-NATIONAL-LEARNING-CAMP-AND-OTHER.pptx
 
Holdier Curriculum Vitae (April 2024).pdf
Holdier Curriculum Vitae (April 2024).pdfHoldier Curriculum Vitae (April 2024).pdf
Holdier Curriculum Vitae (April 2024).pdf
 
This PowerPoint helps students to consider the concept of infinity.
This PowerPoint helps students to consider the concept of infinity.This PowerPoint helps students to consider the concept of infinity.
This PowerPoint helps students to consider the concept of infinity.
 
SOC 101 Demonstration of Learning Presentation
SOC 101 Demonstration of Learning PresentationSOC 101 Demonstration of Learning Presentation
SOC 101 Demonstration of Learning Presentation
 
Basic Civil Engineering first year Notes- Chapter 4 Building.pptx
Basic Civil Engineering first year Notes- Chapter 4 Building.pptxBasic Civil Engineering first year Notes- Chapter 4 Building.pptx
Basic Civil Engineering first year Notes- Chapter 4 Building.pptx
 
Sociology 101 Demonstration of Learning Exhibit
Sociology 101 Demonstration of Learning ExhibitSociology 101 Demonstration of Learning Exhibit
Sociology 101 Demonstration of Learning Exhibit
 
How to Create and Manage Wizard in Odoo 17
How to Create and Manage Wizard in Odoo 17How to Create and Manage Wizard in Odoo 17
How to Create and Manage Wizard in Odoo 17
 

Chapter 4 (maths 3)

  • 1. CHAPTER 4 FOURIER TRANSFORMS INTEGRAL TRANSFORM b The integral transform of a function f (x ) is defined by ∫ f ( x).k (s , x)dx where a k(s , x) is a known function of s and x and it is called the kernel of the transform. When k(s , x) is a sine or cosine function, we get transforms called Fourier sine or cosine transforms. FOURIER INTEGRAL THEOREM If f (x) is a given function defined in (-l , l) and satisfies Dirichlet’s conditions, then ∞ ∞ 1 π ∫ −∫ f ( x) = f (t ) cos λ (t − x) dt dλ 0 ∞ At a point of discontinuity the value of the integral on the left of above equation is 1 { f ( x + 0) − f ( x − 0)}. 2 EXAMPLES 1 for x ≤ 1  1. Express the function f ( x) =  as a Fourier Integral. Hence evaluate 0 for x > 1  ∞ ∞ sin λ cos λx sin λ ∫ λ 0 dλ and find the value of ∫ 0 λ dλ . Solution: We know that the Fourier Integral formula for f (x) is ∞ ∞ 1 f ( x) = π ∫ ∫ f (t ) cos λ (t − x) dt dλ 0 −∞ ……………….(1) Here f (t ) = 1 for t ≤ 1 i.e., f(t) = 1 in -1 < t < 1 f (t ) = 0 for t > 1 f (t ) = 0 in − ∞ < t < −1 and 1 < t < ∞ ∞ 1 1 ∴ Equation (1) ⇒ f ( x) = π ∫ ∫ cos λ (t − x) dt dλ 0 −1 ∞ 1 1  sin λ (t − x)  π ∫ =   dλ 0 λ  −1 ∞ 1 sin λ (1 − x) − sin λ (−1 − x) π∫ = dλ 0 λ ∞ 1 sin λ (1 − x) + sin λ (1 + x ) = ∫ dλ π 0 λ 1
  • 2. 2 sin λ cos λx π∫ ∴ f ( x) = dλ .………………(2) 0 λ [Using sin (A+B) + sin (A-B) = 2 sin A cos B] This is Fourier Integral of the given function. From (2) we get ∞ sin λ cos λx π ∫ 0 λ dλ = 2 f ( x) ……………….(3  1 for x ≤ 1 But f ( x) =  ………………..(4) 0 for x > 1  Substituting (4) in (3) we get π ∞ sin λ cos λx  for x ≤ 1 ∫ λ dλ =  2 0 0 for x > 1  ∞ sin λ π Putting x = 0 we get ∫0 λ dλ = 2 2. Find the Fourier Integral of the function 0 x<0 1  f ( x) =  x=0 2 e − x x > 0  Verify the representation directly at the point x = 0. Solution: The Fourier integral of f (x) is ∞ ∞ 1 f ( x) = π ∫ ∫ f (t ) cos λ (t − x) dt dλ 0 −∞ ……………….(1) 1   ∞ 0 ∞ = ∫ −∫∞ π 0 f (t ) cos λ (t − x )dt + ∫ f (t ) cos λ (t − x)dt d λ 0  1   ∞ 0 ∞ = ∫  ∫ 0. cos λ (t − x)dt + ∫ e −t cos λ (t − x)dt dλ π 0 − ∞ 0  ∞ ∞ 1  e −t = ∫ 2 [ − cos( λt − λx ) + λ sin(λt − λx)]  dλ  π 0 λ +1 0 ∞ 1 cos λx + λ sin λx π∫ f (x) = dλ ……….………(2) 0 λ2 + 1 2
  • 3. Putting x = 0 in (2), we get ∞ 1 f (0) = ∫ 2 1 π 0 λ +1 1 d λ = tan −1 ( λ ) 0 π ∞ [ ] 1 [ = tan −1 ( ∞ ) − tan −1 (0) π ] 1 π  1 =  = π 2 2 1 The value of the given function at x = 0 is . Hence verified. 2 FOURIER SINE AND COSINE INTEGRALS The integral of the form 2∞ ∞ f ( x) = ∫ sin λx ∫ f (t ) sin λt dt dλ π0 0 is known as Fourier sine integral. The integral of the form ∞ ∞ 2 f ( x) = ∫ cos λx ∫ f (t ) cos λt dt dλ π 0 0 is known as Fourier cosine integral. PROBLEMS 1. Using Fourier integral formula, prove that 2(b 2 − a 2 ) ∞ u sin xu ∫ (u 2 + a 2 )(u 2 + b 2 ) du (a, b > 0) − ax − bx e −e = π 0 Solution: The presence of sin xu in the integral suggests that the Fourier sine integral formula has been used. Fourier sine integral representation is given by ∞ ∞ 2 f ( x) = ∫ sin ux ∫ f (t ) sin ut dt du π 0 0 ∞ ∞  sin ux du  ∫ ( e − at − e −bt ) sin ut dt  2 e − ax − e −bx = ∫ π 0 0  3
  • 4. ∞ 2  e − at − bt  = ∫ sin ux du  2 { − a sin ut − u cos ut} − 2e 2 { − b sin ut − u cos ut}  π 0 a + u 2 b +u 0 ∞ 2  u u  = ∫ sin ux du  a 2 + u 2 − b 2 + u 2  π 0   ∞ 2(b 2 − a 2 ) u sin ux = π ∫ (u 2 + a 2 )(u 2 + b 2 ) du 0 2. Using Fourier integral formula, prove that 2 ∞ (λ 2 + 2 ) cos xλ ∫ dλ −x e cos x = π 0 λ2 + 4 Solution: The presence of cos xλ in the integral suggests that the Fourier cosine integral formula for e − x cos x has been used. Fourier cosine integral representation is given by ∞ ∞ 2 π∫ f ( x) = cos λx ∫ f (t ) cos λt dt dλ 0 0 2∞ ∞ − t  ∴e −x cos x = ∫ cos xλ dλ  ∫ e cos t cos λt dt  π 0 0  2 ∞ 1 ∞  = ∫ cos xλ dλ  ∫ e −t { cos(λ + 1)t + cos(λ − 1)t } dt  π 0 2 0  ∞  = 2 π 0  1 −t [ ∫ cos xλ dλ  (λ + 1) 2 + 1 e { − cos(λ + 1)t + (λ + 1) sin(λ + 1)t} ] ∞ 0  + 1 (λ − 1) + 1 2 [ e −t { − cos(λ − 1)t + (λ − 1) sin(λ − 1)t } 0 ∞ ] ) ∞ 1  1 1  = ∫ +  cos xλ dλ π 0  (λ + 1) + 1 (λ − 1) + 1 2 2 ∞ 2 (λ2 + 2) cos xλ = ∫ dλ. π 0 λ2 + 4 COMPLEX FORM OF FOURIER INTEGRALS 4
  • 5. The integral of the form ∞ ∞ 1 ∫ e − iλx ∫ f (t ) e iλt f ( x) = dt d λ 2π −∞ −∞ is known as Complex form of Fourier Integral. FOURIER TRANSFORMS COMPLEX FOURIER TRANSFORMS ∞ 1 The function F [ f ( x)] = ∫∞ f (t ).e dt is called the Complex Fourier transform ist 2π − of f (x ) . INVERSION FORMULA FOR THE COMPLEX FOURIER TRANSFORM ∞ 1 ∫∞F [ f ( x)].e ds is called the inversion formula for the −isx The function f ( x) = 2π − Complex Fourier transform of F [ f ( x)] and it is denoted by F −1 [ F ( f ( x))]. FOURIER SINE TRANSFORMS ∞ 2 The function FS [ f ( x )] = ∫ f (t ).sin st dt is called the Fourier Sine Transform of π 0 the function f (x ) . ∞ 2 The function f ( x) = π ∫ F [ f ( x)]. sin sx ds is called the inversion formula for the 0 S Fourier sine transform and it is denoted by FS −1 [ FS ( f ( x))]. FOURIER COSINE TRANSFORMS ∞ 2 The function FC [ f ( x)] = π ∫ f (t ). cos st dt is called the Fourier Cosine 0 Transform of f (x) . ∞ 2 The function f ( x) = π ∫ F [ f ( x)]. cos sx ds 0 C is called the inversion formula for the Fourier Cosine Transform and it is denoted by FC −1 [ FC ( f ( x))]. PROBLEMS 1. Find the Fourier Transform of 1 − x 2 in x ≤ 1  f ( x) =  0  in x > 1 5
  • 6. sin s − s cos s s 3π Hence prove that ∫ 0 s 3 cos ds = 2 16 . Solution: We know that the Fourier transform of f (x) is given by ∞ 1 F [ f ( x )] = ∫ f ( x).e isx dx 2π −∞ −1 1 ∞ 1 1 1 = ∫ f ( x).e dx + ∫ f ( x).e dx + ∫ f ( x).e isx isx isx dx 2π −∞ 2π −1 2π 1 −1 1 ∞ 1 1 1 = ∫ 0.e dx + ∫ (1 − x ).e dx + ∫ 0.e isx 2 isx isx dx 2π −∞ 2π −1 2π 1 1 1 = ∫ (1 − x 2 ).e isx dx 2π −1 1 1  e isx e isx e isx  =  (1 − x 2 ) − ( −2 x ) 2 2 − 2 3 3  2π  is i s i s  −1 1  − 2 is 2 is − 2 −is 2 e −is  =  2 e + 3e + 2 e −  2π  s is s i s3  1  − 2 is 2  = s (e + e −is ) + 3 (e is − e −is ) 2π  2 is  1 − 4 4  1 4  =  s 2 cos s + s 3 sin s  =  s 3 (sin s − s cos s ) 2π   2π   By using inverse Fourier Transform we get ∞ 1 1 4 f ( x) = 2π . 2π ∫ −∞ s 3 (sin s − s cos s ).e −isx ds ∞ 1 4 = 2π ∫s −∞ 3 (sin s − s cos s ).(cos sx − i sin sx ) ds ∞ 1 4 = 2π ∫ −∞ s3 (sin s − s cos s ) cos sx ds ∞ 1 4 − 2π ∫ −∞ s3 (sin s − s cos s ) i sin sx ds 6
  • 7. The second integral is odd and hence its values is zero. ∞ 2 sin s − s cos s π −∫ ∴ f ( x) = cos sx ds ∞ s3 ∞ 4 sin s − s cos s = ∫ cos sx ds π 0 s3 ∞ sin s − s cos s π i.e., ∫ 0 s 3 cos sx ds = f ( x) 4 1 Putting x = , we get 2 ∞ sin s − s cos s s π  1  π  1  3π ∫0 s 3 cos ds = 2 f   = 1 −  = 4  2  4  4  16 . ∞ sin s − s cos s s 3π ∫ 0 s 3 cos ds = 2 16 . 2. Find the Fourier sine transform of e − x , x ≥ 0 (or) e − x , x > 0. Hence evaluate ∞ x sin mx ∫ 1 + x 2 dx. 0 Solution: The Fourier sine transform of f(x) is given by ∞ 2 FS [ f ( x )] = ∫ f ( x).sin sx dx π 0 −x −x Here e =e for x > 0 ∞ [ ] FS e − x = 2 π ∫e −x . sin sx dx 0 2 s ∞ − ax b  =  ∫ e sin bx dx = 2  π s2 +1 0 a + b2  Using inverse Fourier sine transform we get ∞ ∫ F [e ]. sin sx ds 2 −x f ( x) = s π 0 ∞ 2 2 s = π ∫ 0 . 2 π s +1 . sin sx ds ∞ 2 s = ∫ s 2 + 1 sin sx ds π 0 7
  • 8. π s i.e., f ( x) = ∫ 2 . sin sx ds 2 0 s +1 ∞ s. sin sx π i.e., ∫0 s +1 2 ds = e − x 2 Replacing x by m we get ∞ s. sin ms π i.e., ∫ 0 s +1 2 ds = e − m 2 ∞ x. sin mx π i.e., ∫ 0 x +1 2 dx = e −m 2 [since s is dummy variable, we can replace it by x] e − ax 3. Find the Fourier cosine transform of . x Solution: ∞ 2 We know that FC [ f ( x )] = ∫ f ( x). cos sx dx π 0 − ax e Here f ( x) = . x ∞ 2 e − ax ∴ FC [ f ( x )] = ∫ x . cos sx dx π 0 Let FC [ f ( x)] = F ( s ) ∞ 2 e − ax Then F (s) = π ∫ x . cos sx dx 0 ………………(1) Differentiating on both sides w.r.t. ‘s’ we get, ∞ dF ( s ) d 2 e − ax ds = ds π ∫ x . cos sx dx 0 ∞ 2 ∂  e −ax  = ∫ ∂s  x . cos sxdx π0   ∞ ∞ 2 e −ax 2 − ax = ∫ x (− sin sx).x dx = − π ∫ e sin sx dx π0 0 dF ( s ) 2 s  ∞ b  ∫e − ax =− . 2  sin bx dx =  ds π a + s2  0 a + b2  2 Integrating w.r.t. ‘s’ we get 8
  • 9. 2 s F (s) = − .∫ 2 ds π s + a2 2 1 1 =− . . log ( s 2 + a 2 ) = − . log ( s 2 + a 2 ) π 2 2π e − ax − e −bx 4. Find the Fourier cosine transform of . x Solution: We know that the Fourier cosine transform of f(x) is ∞ 2 FC [ f ( x )] = ∫ f ( x). cos sx dx π 0 − ax e − e −bx Here f ( x) = x ∞  e − ax − e −bx  2 e − ax − e − bx ∴ FC   x =  π ∫ x . cos sx dx 0 ∞ ∞ 2 e − ax 2 e −bx = π ∫ x . cos sx dx − π ∫ x cos sx dx 0 0  e −ax   e −bx  = Fc   − Fc    x   x  1 1 =− log ( s 2 + a 2 ) + log ( s 2 + b 2 ) 2π 2π 1  s2 + b2  = log  2  s + a2  2π   e − as 5. Find f (x) , if its sine transform is . Hence deduce that the inverse sine s 1 transform of . s Solution: We know that the inverse Fourier sine transform of FS [ f (x )] is given by ∞ 2 f ( x) = π ∫ F [ f ( x)]. sin sx ds 0 S e − as Here FS [ f ( x )] = s 9
  • 10. 2 e − as ∴ f ( x) = π ∫ s . sin sx ds 0 Differentiating w.r.t. ‘x’ on both sides, we get, d [ f ( x )] ∞ 2 e − as ∂ dx = π ∫ s . ∂x (sin sx) ds 0 ∞ ∞ 2 e − as 2 − as = π ∫ s . cos sx s ds = π ∫ e . cos sx ds 0 0 2 a  ∞ −ax a  =  ∫ e cos bx dx = 2  π a + x2 2  0 a + b2  d [ f ( x )] 2 a = dx π x + a2 2 2 1 2 1 −1  x  ∴ f ( x) = a π ∫ x 2 + a 2 dx = a π a tan  a    2  x ∴ f ( x) = tan −1   π a 1 To find the inverse Fourier sine transform of : s Put a = 0, in (1), we get 2 2 π π f ( x) = tan −1 (∞) = . = π π 2 2 PROPERTIES 1. Linearity Property If F(s) and G(s) are the Fourier transform of f (x ) and g (x) respectively then F [ a f ( x ) + b g ( x)] = a F ( s) + b G ( s ) Proof: ∞ 1 F [ a f ( x) + b g ( x)] = ∫∞[ a f ( x) + b g ( x)] e dx isx 2π − ∞ ∞ 1 1 = ∫ a f ( x).e dx + ∫ b g ( x).e isx isx dx 2π −∞ 2π −∞ ∞ ∞ a b = ∫ f ( x).e isx dx + ∫ g ( x).e isx dx 2π −∞ 2π −∞ = a F ( s ) + b G ( s) 10
  • 11. 2. Change of Scale Property 1 s If F(s) is the Fourier transform of f (x ) then F [ f (ax)] = F  ,a>0 a a Proof: ∞ 1 F [ f (ax)] = ∫ f (ax).e isx dx 2π −∞ Put ax = y dy a dx = dy i.e., dx = a When x = −∞, y = −∞ and x = ∞, y=∞ ∞ y ∞ s 1 dy 1 1 i  y F [ f (ax)] = is 2π ∫ f ( y).e −∞ a . = a a 2π ∫ f ( y).e −∞ a .dy 1 = F ( s a) a 3. Shifting Property ( Shifting in x ) If F(s) is the Fourier transform of f (x ) then F [ f ( x − a )] = e ias F ( s ) Proof: ∞ 1 F [ f ( x − a)] = ∫ f ( x − a).e isx dx 2π −∞ Put x-a = y dx = dy When x = −∞, y = −∞ and x = ∞, y=∞ ∞ ∞ 1 e ias F [ f ( x − a)] = ∫ f ( y ).e is ( y + a ) . dy = ∫ f ( y).e isy .dy 2π −∞ 2π −∞ ∞ e ias = ∫ f ( x).e .dx = e isa F ( s ) isx 2π −∞ 4. Shifting in respect of s If F(s) is the Fourier transform of f (x ) then F e iax f ( x) = F ( s + a ) [ ] Proof: ∞ Fe[ iax ] f ( x) = 1 ∫e iax f ( x) e isx dx 2π −∞ 11
  • 12. 1 ∫ f ( x).e i( s+a) x = dx = F ( s + a) 2π −∞ 5. Modulation Theorem 1 If F(s) is the Fourier transform of f (x ) then F [ f ( x) cos ax ] = [ F ( s + a ) + F ( s − a )] 2 Proof: ∞ 1 F [ f ( x) cos ax ] = ∫ f ( x). cos ax.e isx dx 2π −∞ ∞ 1  e iax + e −iax  = ∫ 2π −∞ f ( x).e isx    2 dx   ∞ ∞ 1 1 1 1 ∫∞ f ( x).e dx + 2 . 2π ∫ f ( x).e i( s+a ) x i ( s −a ) x = . dx 2 2π − −∞ 1 1 1 = f ( s + a ) + f ( s − a ) = [ f ( s + a ) + f ( s − a )] 2 2 2 1 F [ f ( x) cos ax ] = [ F ( s + a ) + F ( s − a)] 2 COROLLARIES 1 (i ) FC [ f ( x) cos ax ] = [ FC ( s + a) + FC ( s − a)] 2 1 (ii ) FC [ f ( x) sin ax ] = [ FS (a + s) + FS (a − s )] 2 1 (iii ) FS [ f ( x) cos ax ] = [ FS ( s + a ) + FS ( s − a )] 2 1 (iv ) FS [ f ( x) sin ax ] = [ FC ( s − a ) − FC ( s + a)] 2 6. Conjugate Symmetry Property If F(s) is the Fourier transform of f (x ) then F f ( − x) = F ( s) [ ] Proof: ∞ 1 We know that F ( s ) = ∫ f ( x). e isx dx 2π −∞ Taking complex conjugate on both sides we get ∞ 1 ∫∞ f ( x). e dx −isx F (s) = 2π − 12
  • 13. Put x = -y dx = -dy When x = −∞, y = ∞ and x = ∞, y = −∞ −∞ 1 ∴ F (s) = ∫ f (− y) .e (− dy ) isy 2π ∞ −∞ 1 =− ∫ f (− y). e isy dy 2π ∞ [ ] ∞ 1 = ∫ f (− x). e dx = F f (− x) isx 2π −∞ 7. Transform of Derivatives If F(s) is the Fourier transform of f (x ) and if f (x) is continuous, f ′(x) is piecewise continuously differentiable, f (x ) and f ′(x) are absolutely integrable in (−∞ , ∞) and lim [ f ( x)] = 0 , then x → ±∞ F ( f ′( x ) ) = −is F ( s ) Proof: By the first three conditions given, F { f (x)} and F { f ′(x)} exist. ∞ 1 F { f ′( x)} = ∫ f ′( x) e isx dx 2π −∞ ∞ = 1 [e isx ] ∞ f ( x) −∞ − is ∫e isx f ( x) dx, on int egrating by parts. 2π 2π −∞ = 0 − isF { f ( x)} , by the given condition. = −is F ( s). The theorem can be extended as follows. If f , f ′, f ′′, , f ( n −1) are continuous, f (n ) is piecewise continuous, f , f ′, f ′′, , f ( n ) are absolutely integrable in (−∞ , ∞) and f , f ′, f ′′,  , f ( n −1) → 0 as x → ±∞ , then F ( f ( n ) ( x) ) = (−is ) n F ( s ) 13
  • 14. 8. Derivatives of the Transform dF ( s ) If F(s) is the Fourier transform of f (x ) then F [ x. f ( x )] = (−i ) ds Proof: ∞ 1 F (s) = ∫ f ( x )e isx dx 2π −∞ ∞ ∫ ds [ f ( x)e ]dx dF ( s ) 1 d ∴ = isx ds 2π −∞ ∞ i = ∫ [ x. f ( x)]e dx = iF [ xf ( x)] isx 2π −∞ dF ( s) ∴ ( −i ) = F [ x. f ( x)] ds [ ] Extending, we get, F x n . f ( x) = (−i ) n d n F (s) ds n DEFINITION ∞ 1 ∫ f ( x − u ) g (u )du is called the convolution product or simply the convolution 2π −∞ of the functions f (x) and g (x) and is denoted by f ( x) * g ( x ) . 9. Convolution Theorem If F(s) and G(s) are the Fourier transform of f (x ) and g (x) respectively then the Fourier transform of the convolution of f(x) and g(x) is the product of their Fourier transforms. i.e., F [ f ( x ) * g ( x)] = F ( s ).G ( s) Proof ∞ 1 F [ f ( x ) * g ( x )] = ∫∞ f ( x) * g ( x)e dx isx 2π − 1  1 ∞ ∞  isx = ∫∞  2π −∫∞ f ( x − u) g (u )du  e dx 2π −   1 ∞  1 ∞  = ∫∞  2π −∫∞ f ( x − u)e dxdu, isx g (u )  2π −  on changing the order of int egration. 14
  • 15. ∫ g (u )[e ] 1 = ius F ( s) du , by the shifting property. 2π −∞ ∞ 1 = F ( s). ∫ g (u ).e ius du 2π −∞ = F ( s).G ( s ) Inverting, we get F −1 [ F ( s ).G ( s)] = f ( x) * g ( x ) = F −1 { F ( s )} * F −1 { G ( s )} 10. Parseval’s Identity (or) Energy Theorem If f (x) is a given function defined in (−∞ , ∞) then it satisfy the identity, ∞ ∞ ∫ ∫ F ( s) 2 2 f ( x) dx = ds −∞ −∞ where F(s) is the Fourier transform of f (x ) . Proof: We know that F −1 [ F ( s ).G ( s)] = f ( x) * g ( x ) ∞ ∞ 1 1 ∫ F (s).G(s)e ∫ f (t ) g ( x − t )dt −isx ds = 2π −∞ 2π −∞ Putting x = 0, we get ∞ ∞ ∫ F (s).G(s)ds = ∫ f (t ) g (−t )dt −∞ −∞ ………………..(1) Let g ( − t ) = f (t ) .……………….(2) i.e., g (t ) = f (−t ) ………………..(3) ∴ G ( s ) = F [ f (− x)] = F ( s ) by property (9) i.e., ∴ G ( s) = F ( s) ………………..(4) Substituting (2) and (4) in (1) we get ∞ ∞ ∫ F (s).F (s) ds = −∞ ∫ f (t ). f (t ) dt −∞ [ F (s).F (s) = F (s) ] ∞ ∞ ∫ ∫ 2 2 2 F ( s) ds = f ( x) dx −∞ −∞ 11. If f (x) and g (x) are given functions of x and FC [ f ( x)] and FC [ g ( x)] are their Fourier cosine transforms and FS [ f ( x)] and FS [ g ( x)] are their Fourier sine transforms then 15
  • 16. ∞ ∞ (i) ∫ 0 f ( x ) g ( x)dx = ∫ FC [ f ( x)].FC [ g ( x)]ds = ∫ FS [ f ( x )].FS [ g ( x)]ds 0 0 ∞ ∞ ∞ ∫ f ( x) dx = ∫ FC [ f ( x)] ds = ∫ FS [ f ( x )] ds , 2 2 2 (ii) 0 0 0 which is Parseval’s identity for Fourier cosine and sine transforms. Proof: ∞ ∞  2∞  (i) ∫ FC [ f ( x )].FC [ g ( x )]ds = ∫ FC [ f ( x)]  ∫ g ( x) cos sx dx  ds 0 0  π 0  ∞  2∞  = ∫ g ( x)  ∫ FC [ f ( x)] cos sx ds  dx, 0  π 0  Changing the order of integration ∞ = ∫ f ( x) g ( x)dx 0 Similarly we can prove the other part of the result. (ii) Replacing g ( x) = f * ( x) in (i) and noting that FC [ f ( x)] = FC [ f ( x)] and FS [ f ( x)] = FS [ f ( x)] , we get ∞ ∞ ∞ ∫0 f ( x ). f ( x).dx = ∫ FC [ f ( x)].FC [ f ( x)] ds = ∫ FS [ f ( x )].FS [ f ( x )] ds 0 0 ∞ ∞ ∞ ∫ f ( x) .dx = ∫ FC [ f ( x)] ds = ∫ FS [ f ( x)] ds 2 2 2 i.e., 0 0 0 12. If FC [ f ( x)] = FC ( s ) and FS [ f ( x)] = FS ( s ) , then d (i) { FC ( s)} = − FS { xf ( x)} and ds d (ii) { FS ( s)} = − FC { xf ( x)}. ds Proof: ∞ 2 π∫ FC ( s) = f ( x) cos sx dx 0 ∞ d { FC ( s)} = ∫ f ( x)(− x sin sx)dx ds 0 ∞ = − ∫ {xf ( x)}sin sx dx 0 = − FS {xf ( x)} Similarly the result (ii) follows. 16
  • 17. PROBLEMS a 2 − x 2  x <a 1. Show that the Fourier transform of f ( x ) =  is 0  x >a>0 ∞ 2  sin as − as cos as  sin t − t cos t π 2   . Hence deduce that ∫ dt = . Using Parseval’s π s 3  0 t 3 4 ∞ 2  sin t − t cos t  π identity show that ∫  3  dt = . 0  t 15 Solution: We know that ∞ 1 F [ f ( x )] = ∫ f ( x).e isx dx 2π −∞ 1   −a a ∞ =  ∫ f ( x)e dx + ∫ f ( x)e dx + ∫ f ( x )e dx  isx isx isx 2π  −∞ −a a  1   a a 1 =  0 + ∫ ( a 2 − x 2 )e isx dx + 0 = ∫ (a 2 − x 2 ).e isx dx 2π  − a  2π −a a 1  2  e isx   e isx   e isx  =  ( a − x 2 )  is  − (−2 x) 2 2  − 2 3 3   i s  i s  2π        − a = 1  − 2a isa  2 e +e [−isa 2i ] [  + 3 e −isa − e isa  ] 2π  s s  1  − 2a 4  =  2 [2 cos as ] + 3 sin as  2π  s s  4  sin as − as cos as  = 2π   s3   2  sin as − as cos as  F (s) = 2 π  s3   2  sin s − s cos s  When a = 1, F ( s ) = 2 ………………..(A) π   s3   Using inverse Fourier Transform, we get 17
  • 18. 1 2 1 f ( x) = 2π .2. π ∫s −∞ 3 [sin as − as cos as].e −isx ds ∞ 1 2 1 { sin as − as cos as}{ cos sx − i sin sx} ds π −∫ s 3 = .2. 2π ∞ ∞ 2 sin as − as cos as π −∫ f ( x) = cos sx ds ∞ s3 [The second integral is odd and hence its value is zero] ∞ 4 sin as − as cos as = ∫ cos sx ds π 0 s3 [since the integrand is an even function of s] Putting a = 1, we get ∞ 4 sin s − s cos s f ( x) = ∫ cos sx ds π 0 s3 ∞ 4 sin t − t cos t f ( x) = ∫ cos tx dt π 0 t3 Putting x = 0, in the given function we get ∞ 4 sin t − t cos t π∫ dt = f (0) = 1 0 t3 ∞ sin t − t cos t π ∴ ∫ 0 t 3 dt = 4 ∞ ∞ ∫ ∫ F (s) 2 2 Using Parseval’s identity, f ( x) dx = ds [Using (A)] −∞ −∞ 2  2  ∞  2. (sin s − s cos s)  1 π ∫  − ∞ s3  ds = ∫ (1 − x 2 ) 2 dx  −1     ∞ 2 1 8  sin s − s cos s  ∫∞  ds = 2 ∫ (1 − x ) dx 2 2 π− s 3  0 ∞ 2 16  sin s − s cos s  8 ∫ π 0 s 3  ds = 2.  15 18
  • 19. 2  sin s − s cos s  π i.e., ∫ 0 s 3  ds =  15 ∞ 2  sin t − t cos t  π i.e., ∫ t3 0  dt =  15 2. Find the Fourier Transform of f (x) if 1 − x , x < 1  f ( x) =  0,  x >1 ∞ 4  sin t  π Hence deduce that ∫   dt = 0 t  3 Solution: We know that ∞ 1 F [ f ( x )] = ∫∞ f ( x).e dx isx 2π − 1 ∫ (1 − x ).e 1 = isx dx 2π −1 Since x > 1, f ( x) = 0, i.e., in − ∞ < x < −1, and 1 < x < ∞, f ( x ) = 0. 1 ∫ [1 − x ] [ cos sx + i sin sx] dx 1 = 2π −1 1 ∫ [1 − x ] cos sx dx 1 = 2π −1 The second integral becomes zero since it is an odd function. 1 2 = 2π ∫ (1 − x) cos sx dx 0 [ [1 − x ] cos x is an even function] 1 2   sin sx   − cos sx  = (1 − x ) s  − (−1) s 2  π      0 2  − cos s 1  =  + 2 π  s2 s  2 1 i.e., F ( s) = . (1 − cos s ) π s2 19
  • 20. Using Parseval’s identity ∞ ∞ ∫ ∫ F ( s) 2 2 f ( x) dx = ds −∞ −∞ 1 ∞ [1 − x ] dx = 2 ∫ (1 − cos s) ds 2 ∫ 2 −1 π −∞ s4 1 ∞ 2 (1 − cos s) 2 2 ∫ [1 − x ] dx = π −∫ 2 ds 0 ∞ s4 ∞ 2 2 (1 − cos s ) 2 3 π −∫ = ds ∞ s4 put s = 2t when s = ∞, t = ∞ ds = 2dt when s = −∞, t = −∞ π ∞ (1 − cos 2t ) 2 3 −∫ = .2 dt ∞ 16t 4 π ∞ (1 − cos 2t ) 2 3 −∫ = dt ∞ 8t 4 ∞ π (1 − cos 2t ) 2 = 2∫ dt 3 0 8t 4 π ∞ sin 4 t 3 ∫ t4 = dt 0 ∞ 4  sin t  π i.e., ∫  t  dt = 3 0  ∞ dx 3. Evaluate ∫ (x 0 2 + a )( x 2 + b 2 ) 2 using transforms. Solution: 2 a We know that the Fourier cosine transform of f ( x) = e − ax is . 2 . π s + a2 2 b Similarly the Fourier cosine transform of f ( x) = e − ax is . 2 . π s + b2 20
  • 21. ∞ We know that ∫ FC [ f ( x)].FC [ g ( x)] ds = ∫ f ( x).g ( x) dx 0 0 ∞ ∞ 2 a 2 b i.e., ∫ 0 . 2 . . 2 π s +a π s +b 2 2 . ds = ∫ e − ax .e −bx dx 0 ∞ ∞ 2 ab ∫ (s 2 + a 2 )(s 2 + b 2 ) ds = ∫ e −( a +b ) x i.e., ds π 0 0 ∞  e −( a + b ) x  1 1 =  = 0− =  − ( a + b)  0 − ( a + b) a + b ∞ dx π i.e., ∫ (x 0 2 2 2 2 = + a )( x + b ) 2ab( a + b) 4. Find the Fourier transform of e − a x and hence deduce that ∞ cos xt π −a x (i) ∫ 2 2 dt = e 0 a +t 2a [ (ii) F xe −a x ]=i 2 2as π (s + a 2 ) 2 2 Solution: ∞ 1 F [ f ( x )] = ∫ f ( x).e isx dx 2π −∞ 1   0 ∞ = ∫ f ( x)e isx dx + ∫ f ( x)e isx dx  2π  −∞ 0  e − ax  if 0 ≤ x < ∞ Here f ( x ) =  ax e  if − ∞ < x < 0 1  ax isx  0 ∞ = ∫ e. e dx + ∫ e −ax .e isx dx  2π  −∞ 0  1  ( a +is ) x  0 ∞ =  ∫ e. dx + ∫ e −( a −is ) x dx  2π  −∞ 0  21
  • 22. 1  e ( a +is ) x   e −( a −is ) x   0 ∞ =   +   2π  (a + is)  −∞  − (a − is )  0    1  1 1  =  a + is + a − is  2π   Fe[ −a x ]= 2 a π s + a2 2 Using inversion formula, we get ∞ 1 2 a f ( x) = 2π ∫ −∞ . 2 π s +a 2 e −isx ds ∞ a cos sx − i sin sx π −∫ = ds ∞ s2 + a2 ∞ a cos x = π−∫∞ s 2 + a 2 ds ∞ cos sx π π −a x ∫s 0 2 +a 2 dx = 2a f ( x) = 2a .e (or ) ∞ cos tx π −a x ∫s 0 2 +a 2 dt = 2a .e Putting a = 1, we get, Fe[ ]=−x 2 1 . π s2 +1 ∞ ∞ cos sx π −x cos tx π −x and ∫ s 2 + 1 ds = 2 e 0 (or ) ∫t 0 2 +1 dt = e 2 FINITE FOURIER TRANSFORMS If f (x) is a function defined in the interval (0 , l) then the finite Fourier sine transform of f (x) in 0 <x < l is defined as 22
  • 23. nπx l FS [ f ( x)] = ∫ f ( x). sin dx where ‘n’ is an integer 0 l The inverse finite Fourier sine transform of FS [ f (x )] is f (x) and is given by 2 ∞ nπx f ( x) = ∑ FS [ f ( x )] sin l n =1 l The finite Fourier cosine transform of f (x ) in 0 < x < l is defined as nπx l FC [ f ( x)] = ∫ f ( x). cos dx where ‘n’ is an integer 0 l The inverse finite Fourier cosine transform of FC [ f (x)] is f (x) and is given by 1 2 ∞ nπx f ( x) = FC (0) + ∑ FC [ f ( x )] cos l l n =1 l PROBLEMS 1. Find the finite Fourier sine and cosine transforms of f ( x) = x 2 in 0 < x < l. Solution: The finite Fourier sine transform is nπx l FS [ f ( x)] = ∫ f ( x). sin dx 0 l Here f ( x) = x 2 [ ] nπx l FS x 2 = ∫ x 2 . sin dx 0 l l   nπx   nπx   nπx    − cos   − sin   cos  = x 2 l  − 2 x l  + 2 l    nπ   n 2π 2   n 3π 3           l   l2   l 3  0  − l3 2l 3 2l 3 = cos nπ + 3 3 cos nπ − 3 3 nπ nπ nπ = l3 nπ 2l 3 [ (−1) n +1 + 3 3 (−1) n − 1 nπ ] The finite Fourier cosine transform is 23
  • 24. nπx l FC [ f ( x)] = ∫ f ( x). cos dx 0 l Here f ( x) = x 2 [ ] nπx l FC x 2 = ∫ x 2 . cos dx 0 l l   nπx   nπx   nπx    sin   − cos   − sin  = x 2  l  − 2 x l  + 2 l    nπ   n 2π 2   n 3π 3    l         l2   l3  0 2l 3 = cos nπ n 2π 2 2l 3 = (−1) n nπ 2 2 2. Find the finite Fourier sine and cosine transforms of f ( x ) = x in (0 , π ) . Solution: The finite Fourier sine transform of f ( x) = x in (0 , π ) is π FS [ f ( x)] = ∫ f ( x ). sin nx dx 0 Here f ( x) = x in (0 , π ) π π   − cos nx   − sin nx  FS [ x ] = ∫ x. sin nx dx =  x  − 1 2  0   n   n  0 π π =− cos nπ = (−1) n +1 . n n The finite Fourier cosine transform of f ( x) = x in (0 , π ) is π FC [ f ( x)] = ∫ f ( x). cos nx dx 0 Here f ( x) = x in (0 , π ) π π   sin nx   − cos nx  FC [ x ] = ∫ x. cos nx dx =  x  − 1 2  0   n   n  0 = 1 n 2 1 1 [ cos nπ − 2 = 2 (−1) n − 1 n n ] 24
  • 25. 2π (−1) p−1 3. Find f (x) if its finite sine transform is given by , where p is positive p3 integer and 0 < x < π . Solution: We know that the inverse Fourier sine transform is given by ∞ 2 f ( x) = ∑ FS [ f ( x )] sin px ………………..(1) π p =1 2π (−1) p−1 Here FS [ f (x )] = ………………..(2) p3 Substituting (2) in (1), we get 2 ∞ 2π (−1) p −1 f ( x) = ∑ sin px π p =1 p3 ∞ (−1) p −1 = 4∑ sin px p =1 p3  2 pπ  cos   3  find FC [ f ( p )] if 0 < x <1. −1 4. If f ( p) = (2 p + 1) 2 Solution: ∞ nπx We know that FC −1 [ f ( p)] = 1 FC (0) + 2 ∑ FC [ f ( x)] cos l l n =1 l  2 pπ  cos  Here f ( p) =  3  (2 p + 1) 2 Let FC [ f ( x)] = f ( p) ∞ nπx ∴ FC −1 [ f ( p)] = 1 f C ( 0) + 2 ∑ f ( p ) cos [ l = 1] l l n =1 l  2 pπ  ∞ cos  = 1 + 2∑  3  . cos nπx n =1 (2 p + 1) 2 25
  • 26. UNIT-4 PART A 1. State the Fourier integral theorem. Ans: If f (x) is a given function defined in (-l , l) and satisfies Dirichlet’s conditions, then ∞ ∞ 1 f ( x) = π ∫ ∫ f (t ) cos λ (t − x) dt dλ 0 −∞ 2. State the convolution theorem of the Fourier transform. Ans: If F(s) and G(s) are the Fourier transform of f (x ) and g (x) respectively then the Fourier transform of the convolution of f(x) and g(x) is the product of their Fourier transforms. i.e., F [ f ( x ) * g ( x)] = F ( s ).G ( s) 3. Write the Fourier transform pair. Ans: F [ f (x)] and F −1 [ F ( S )] are Fourier transform pairs. 4. Find the Fourier sine transform of f ( x) = e − ax (a > 0). Ans: ∞ 2 FS [ f ( x)] = π ∫ f ( x ). sin sx dx 0 2 ∞ 2 s   ∞ −ax b  ∫ e . sin sx dx =  ∫ e sin bx dx = 2 − ax =  π 0 π  s2 + a2     0 a + b2  2 s  = π  s2 + a2    5. If the Fourier transform of f (x ) is F(s) then prove that . F [ f ( x − a )] = e isa F ( s) Ans: ∞ 1 F [ f ( x − a)] = ∫∞ f ( x − a).e dx isx 2π − Put x-a = y dx = dy When x = −∞, y = −∞ and x = ∞, y = ∞ ∞ ∞ 1 e ias F [ f ( x − a)] = ∫ f ( y).e is ( y + a ) . dy = ∫ f ( y).e isy .dy 2π −∞ 2π −∞ ∞ e ias = ∫ f ( x).e .dx = e isa F ( s ) isx 2π −∞ 6. State the Fourier transforms of the derivatives of a function. 26